数学 已知ai≡bi(mod m)证明a1a2···ak≡b1b2···bk(

扫二维码下载作业帮
2亿+学生的选择
下载作业帮安装包
扫二维码下载作业帮
2亿+学生的选择
完全剩余系m为奇数,{a1,a2...am}与{b1,b2...bm}都是模m的完全剩余系,且ai≡bi(mod m).证明:{a1+b1,a2+b2...am+bm}也是模m的完全剩余系.是证明{a1b1,a2b2,...ambm}不能构成模m的完全剩余系
ree挚爱00308
扫二维码下载作业帮
2亿+学生的选择
楼上关于第一问的证明是正确的.第二问:{a1b1,a2b2,...ambm}等价于{0,1^2,2^2,3^2.(m-1)^2},假设其中两个元素关于m同余,即(m-k)^2-(m-n)^2 整除m,其中k=!n,1=
为您推荐:
其他类似问题
只要证明{0,2,4,...,2(m-1)}构成模m的完全剩余系即可,这是因为{0,2,4,...,2(m-1)}就是{a1+b1,a2+b2...am+bm}。反证假设,如果在集合{0,2,4,...,2(m-1)}中有两个元素模m同余,不妨设为2x和2y模m同余(0<=y<x<m),则2x-2y能被m整除。注意到m为奇数,则x-y也能被m整除。但是由0<=y<x<m可知,0...
扫描下载二维码&#xe621; 上传我的文档
&#xe602; 下载
&#xe60c; 收藏
该文档贡献者很忙,什么也没留下。
&#xe602; 下载此文档
正在努力加载中...
离散数学 (数论基础篇二)
下载积分:2500
内容提示:离散数学 (数论基础篇二)
文档格式:PPT|
浏览次数:41|
上传日期: 02:45:41|
文档星级:&#xe60b;&#xe60b;&#xe60b;&#xe60b;&#xe60b;&#xe60b;
全文阅读已结束,如果下载本文需要使用
&#xe71b; 2500 积分
&#xe602;下载此文档
该用户还上传了这些文档
离散数学 (数论基础篇二)
关注微信公众号中国数学奥林匹克(CMO)历届试题及解答()_学霸学习网
中国数学奥林匹克(CMO)历届试题及解答()
中国 数 学奥 林 匹 克 (CMO) 历届试题及解答 第一届中国数学奥林匹克(1986年)天津 南开大学1.已知 a1 , a2 , . . . , an 为实数, 如果它们中任意两数之和非负,那么对于满足 x1 + x2 + ? ? ? + xn = 1 的任意非负实数 x1 , x2 , . . . , xn , 有不等式 a1 x1 + a2 x2 + ? ? ? + an xn 成立.请证明上述命题及其逆命题. 证明:原命题的证明:由0 (1)若ai 以ai + a1 xi 1, xi ? x2 i 0, xi x2 i (i = 1, 2, . . . , n).2 2 a1 x2 1 + a2 x2 + ? ? ? + 2 2 a1 x2 1 + a2 x2 + ? ? ? + an xn0(i = 1, 2, . . . , n),则显然有a1 x1 + a2 x2 + ? ? ? + an xn 0, ai ?a1 & 0(i = 2, 3, . . . , n). ∴(2)否 则 至 少 存 在 一 个ai & 0,由 对 称 性 不 妨 设a1 & 0. 又 因 为a1 , a2 , . . . , an 中 任 两 数 之 和 非 负,所2 2 a1 x1 + a2 x2 + ? ? ? + an xn ? a1 x2 1 ? a2 x2 ? ? ? ? ? an xn 2 2 = a1 (x1 ? x2 1 ) + a2 (x2 ? x2 ) + ? ? ? + an (xn ? xn ) 2 2 a1 (x1 ? x2 1 ) + (?a1 )(x2 ? x2 ) + ? ? ? + (?a1 )(xn ? xn ) 2 2 = (?a1 )(x2 1 ? x2 ? ? ? ? ? xn ? x1 + x2 + ? ? ? + xn ) 2 2 = (?a1 )(x2 1 ? x1 + (1 ? x1 ) ? x2 ? ? ? ? ? xn ) 2 = (?a1 )((1 ? x1 )2 ? x2 2 ? ? ? ? ? xn ) 2 = (?a1 )((x2 + ? ? ? + xn )2 ? x2 2 ? ? ? ? ? xn )02 x2 2 + ? ? ? + xn . 1 4 (ai2 最后一步是由于x2 , x3 , . . . , xn & 0, (x2 + ? ? ? + xn )2 = x2 2 + ? ? ? + xn +逆命题的证明:对于任意的1 ∴ ai + aji&jn,令xi = xj =xi xj 2 i&j n 1 1 2 ,其余xk 均等于0.则 2 (ai + aj )+ aj ).0,即任两数之和非负.证毕.2.在三角形ABC 中,BC 边上的高AD = 12,∠A的平分线AE = 13,设BC 边上的中线AF = m,问m在什 么范围内取值时,∠A分别为锐角,直角,钝角? 解:设O为 ABC 的外心,不妨设AB & AC ,∠B 为锐角. 则OF 垂 直 平 分 线 段BC ,由 外 心 的 性 质,∠C 为 锐 角 时,∠OAB = ∠OBA =1 ? 2 (180 1 ? 2 (180? ∠AOB ) =? 2∠C ) = 90? ? ∠C .又因为AD ⊥ BC ,∴ ∠CAD = 90? ? ∠C ,∴ ∠OAB = ∠DAC . 类似地,当∠C 为直角或钝角时也有∠OAB = ∠DAC . 由AE 平分∠BAC ,∠BAE = ∠CAE .∴ ∠OAE = ∠DAE .(由于F, D在E 两侧). ∠A为锐角时,O, A在BC 同侧,∠F AE & ∠OAE = ∠DAE ; ∠A为直角时,O, F 重合,∠F AE = ∠OAE = ∠DAE ; ∠A为钝角时,O, A在BC 异侧,∠F AE & ∠OAE = ∠DAE . 1 √ sin ∠F AE FE AD 由正弦定理 sin AE 2 ? AD2 = 5, ∠DAE = DE × AF .其中DE = √ √ F E = F D ? DE = AF 2 ? AD2 ? DE = m2 ? 122 ? 5 & 0. ∴ m & 13, 且∠A为锐角等价于 ∠A为直角等价于 ∠A为钝角等价于 解得当13 & m & 当m = 当m &√ √ √ m2 ?122 ?5 5×12 m 12 m12 m& 1;m2 ?122 ?5 5 m2 ?122 ?5 5× ×= 1; & 1. 时,∠A为锐角; 时,∠A为直角;
时,∠A为钝角.3.设z1 , z2 , . . . , zn 为复数,满足 |z1 | + |z2 | + ? ? ? + |zn | = 1. 求证:上述n个复数中,必存在若干个复数,它们的和的模不小于 1 6. 证明:设zk = xk + yk i(xk , yk ∈ R, k = 1, 2 . . . , n) 将所有的zk 分为两组X,Y.若|xk | 再将X中的复数分为两组A,B.若xk 则zk ∈A|yk |,则将zk 放入X中;若|yk | 0,则将zk 放入A中;若xk1 4.|xk |,则将zk 放入Y中. 其中必有一组中 0,则将zk 放入B中. 其中必有一组中的所有复数模长之和不小于 1 2 .不妨设为X.1 所有复数摸长之和不小于 4 .不妨设为A.|zk |而对于zk ∈ ∴ xkzk ∈A 2 , yk A,x2 k 4 2 1 √1 4 ,即2 x2 k + yk 2 x2 k + yk√2xk . yk |zk ∈A zk ∈A.∴ |zk ∈Azk | = |zk ∈A 1 6.xk + ixkzk ∈A √ 而4 2 & 6, ∴ |1 √ . 4 2zk |zk ∈A即A中复数之和的模不小于 1 6 .证毕. 另证:设zk = xk + yk i(xk , yk ∈ R, k = 1, 2 . . . , n) 则|zk | = ∴ ∴|xk 0 n k=1 2 x2 k + yk|xk | + |yk |. 1. xk | + |yk 0|xk | + |yk | xk | + |xk &0yk | + |yk &0yk |1. xk |1 4.其中必有一项不小于 1 4 ,不妨设为第一项,则 | ∴|xk 0xk 0 1 4zk | = |xk 0xk + ixk 0yk ||xk 0xk |&1 6 .证毕.4.已知:四边形P1 P2 P3 P4 的四个顶点位于三角形ABC 的边上. 求证:四个三角形 P1 P2 P3 , 的四分之一. 证明:有两种情况:(1)四个顶点在两条边上;(2)四个顶点在三条边上. (1)不妨设P1 , P4 在AB 上,P2 , P3 在AC 上,P1 , P2 分别在AP4 , AP3 上. 将B 移至P4 ,C 移至P3 ,三角形ABC 的 2 P1 P2 P4 , P1 P3 P4 , P2 P3 P4 中,至少有一个的面积不大于 ABC 的面积 面积减小,归为情形(2). (2)不妨设P1 在AB 上,P2 在AC 上,P3 , P4 在BC 上,P3 在P4 C 上. (2.1)若P1 P2 上的高的长度. ∴SP1 P2 P31 BC ,设 AP AB =AP2 AC= λ,P1 P2 = λBC .P1 P2 到BC 的距离为(1?λ)h,h为三角形ABC 中BC 边1 4S ABC .= λ(1 ? λ)SABC(2.2)若P1 P2 不 平 行 于BC ,不 妨 设P1 到BC 的 距 离 大 于P2 到BC 的 距 离. 交AB 于E ,交P1 P4 于D.则S 由(2.1)知SEP2 P3 1 4S P1 P2 P 3 , S P4 P2 P3 中有一个不大于S ABC .则S P1 P 2 P3 , S 1 P4 P2 P3 中有一个不大于 4 S过P2 作 平 行 于BC 的 直 线EP2 P3 . ABC .证毕.DP2 P3 ,也就不大于S5.能否把1,1,2,2,. . . ,这些数排成一行, 使得两个1之间夹着1个数,两个2之间夹着2个数,. . . , 两 个1986之间夹着1986个数.请证明你的结论. 解:不能.假设可以做出这样的排列,将已排好的数按顺序编号为1,2,. . . ,3972. 当n为奇数时,两个n的编号奇偶性相同;当n为偶数时,两个n的编号奇偶性不同. 而1到1986之间有993个 偶数,所以一共有2k + 993个编号为偶数的数.(k ∈ N? ) 但是1到3972之间有1986个偶数,k = 496.5.矛 盾.所以不能按要求排成这样一行. √ 6.用任意的方式,给平面上的每一点染上黑色或白色. 求证:一定存在一个边长为1或 3的正三角形,它的三个顶点是同色的. 证明:(1)若平面上存在距离为2的两个点A, B 异色,设O为它们的中点,不妨设A, O同色. 考虑以AO为一 √ 边的正三角形AOC, AOD,若C, D中有一个与A, O同色,则该三角形满足题意. 否则BCD为边长 3的 同色正三角形. (2)否则平面上任两个距离为2的点均同色,考虑任意两个距离为1的点,以他们连线为底,2为腰长作等腰 三角形,则任一腰的两顶点同色. 所以三个顶点同色,即任两个距离为1的点同色.所以平面上任意一个边 长为1的正三角形三个顶点同色.证毕.3 第二届中国数学奥林匹克(1987年)北京 北京大学1.设n为自然数,求证方程z n+1 ? z n ? 1 = 0有模为1的复根的充分必要条件是 n + 2可被6整除. 证明:当6|n + 2时,令z = ei 3 = ∴ z n+1 ? z n ? 1 = e ∴zn+1 n ?i π 3π1 2+√3 6 2 i, z= 1, |z | = 1.√ 3 2 i)?eiπ 3? 1 = (1 2 ?? (? 1 2 ?√3 2 i)? 1 = 0.? z ? 1 = 0有模为1的复根.若z n+1 ? z n ? 1 = 0有模为1的复根eiθ = cos θ + i cos θ. 则z n+1 ? z n ? 1 = (cos(n + 1)θ ? cos nθ ? 1) + i(sin(n + 1)θ ? sin nθ) = 0.+1 ∴ cos(n + 1)θ ? cos nθ ? 1 = ?(2 sin 2n2 θ sin θ 2 + 1) = 0. +1 sin(n + 1)θ ? sin nθ = 2 cos 2n2 θ sin θ 2 = 0. +1 +1 1 θ ∴ cos 2n2 θ = 0, sin 2n2 θ = ±1, sin θ 2 = ± 2 , 设 2 = ?. π (1)sin ? = 1 2 ,sin(2n + 1)? = ?1. ? = 2kπ + 6 或2kπ + 5π 6 ,k∈ Z.1 3 2n+1 (2n + 1)? = (2l + 3 = 2t + 3 2 )π (l ∈ Z). ∴ (2n + 1)(2k + 6 ) = 2l + 2 , 6 2 , n = 6t + 4(t ∈ Z). 5(2n+1) 5 ) = 2l + 3 = 2t + 3 或(2n + 1)(2k + 6 2, 6 2 , 5|4t + 3, t ≡ 3 (mod 5)(t ∈ Z).设t = 5s + 3,则n = 6s + 4,总有6|n + 2. (2)sin ? = ? 1 2 ,sin(2n + 1)? = 1.显然以??代?即有(1).所以6|n + 2.证毕. 2.把边长为1的正三角形ABC 的各边都n等分,过各分点平行于其它两边的直线, 将这三角形分成若干个 小三角形,这些小三角形的顶点都称为结点, 并且在每一结点上放置了一个实数.已知: (1)A, B, C 三点上放置的数分别为a, b, c. (2)在每个由有公共边的两个最小三角形组成的菱形之中, 两组相对顶点上放置的数之和相等. 试求:(1)放置最大数的点和放置最小数的点之间的最短距离. (2)所有结点上数的总和S . 解:(1)不难证明同一直线上相邻三个结点上放置的数中间一个为两边的等差中项,所以同一直线上的数 按顺序成等差数列. 若两端的数相等,则所有的数都相等.否则两端的数为最大的和最小的. 若a, b, c相等,显然所有数都相等,最短距离显然为0. 若a, b, c两两不等,最大的数与最小的数必出现在A, B, C 上,最短距离为1. 若a, b, c有两个相等但不与第三个相等,不妨设a = b & c,最小的数为c,最大的数出现在线段AB 的任意 结点上. 当n为偶数时,与C 最近的为AB 中点,最短距离为1 边偏 21 n 的点,最短距离为 2 √ 3 2 .当n为奇数时,与C 最近的为AB 中点向两3+1 n2 .2 π, 4 (2)将这个三角形绕中心旋转 3 3 π 弧度,得到的两个三角形也满足题意(2). 将这三个三角形对应结点的数相加形成的三角形也满足(2),三个顶点上的数均为a + b + c.由(1)的分析知所有结点上的数均1 1 1 为a + b + c. 共 2 (n + 1)(n + 2)个结点,∴ S = 1 3 ( 2 (n + 1)(n + 2))(a + b + c) = 6 (n + 1)(n + 2)(a + b + c).3.某次体育比赛,每两名选手都进行一场比赛, 每场比赛一定决出胜负,通过比赛确定优秀选手, 选 手A被确定为优秀选手的条件是:对任何其它选手B, 或者A胜B,或者存在选手C,C胜B,A胜C. 结果按上 4 述规则确定的优秀选手只有一名, 求证:这名选手一定胜所有其它选手. 证明:假设该优秀选手为A,且存在其他选手胜A. 设B为所有胜A的人中胜的场次最多的一个,由B不是优秀选手,必存在选手C使得C胜B, 且不存在选 手D使得B胜D,D胜C. 由B胜A,C也胜A,且C胜B胜过的所有人.C至少比B多胜一场,且C胜A,与B的选取 矛盾.所以A胜所有人. 4.在一个面积为1的正三角形内部,任意放五个点,试证:在此正三角形内, 一定可以作三个正三角形盖住 这五个点, 这三个正三角形的各边分别平行于原三角形的边, 并且它们的面积之和不超过0.64.100 证明:可将0.64换成 169 + ε(ε & 0).在面积为1的正三角形ABC 中,在AB 上取A1 , B2 ,AC 上取A2 , C1 ,BC 上取B1 , C2 , 使得AA1 = AA2 = BB1 = BB2 = CC1 = CC2 = (1)若 AB2 C1 , BC2 A1 ,3 13 AB .连结A1 C2 , A2 B1 , B2 C1 交于A0 , B0 , C0 . ε 的 CA2 B1 中有一个至少包含五个点中的三个,另两个点可分别用面积为 2 ε 22 正三角形覆盖, 面积之和为( 10 13 ) + 2 ×=100 169+ ε.100 169(2)菱 形AA1 A0 A2 , BB1 B0 B2 , CC1 C0 C2 中 有 两 个 有 两 个 点,另 一 个 中 有 一 个 点, 则 可 用 两 个 边 长6 6 2 为 13 AB 的正三角形和一个面积为ε的正三角形覆盖. 面积之和为2( 13 ) +ε&+ ε.(3)菱形AA1 A0 A2 , BB1 B0 B2 , CC1 C0 C2 中有两个有一个点,另一个中有两个点, 不妨设为AA1 A0 A2 ,6 AB 的正三角形覆盖AA1 A0 A2 中 则B1 B0 C0 C2 中有一个点,不妨设这个点更靠近B , 则可用一个边长为 13 6 两个点, 用一个边长为 13 AB 的正三角形覆盖BB1 B0 B2 , B1 B0 C0 C2 中的点. 用一个面积为ε的正三角形 6 2 8 2 覆盖最后一个点, 面积之和为( 13 ) + ( 13 ) +ε= 100 169+ ε.证毕.注:当五个点取为A, B, C, A0 , B0 C0 中点是不难证明不能用三个面积之和为 100 169 的正三角形覆盖这五个100 点. 即 169 + ε(ε & 0)为最优.5.设A1 A2 A3 A4 是一个四面体, S1 , S2 , S3 , S4 分别是以A1 , A2 , A3 , A4 为球心的球, 它们两两相外切.如果 存在一点O, 以这点为球心可作一个半径为r的球与S1 , S2 , S3 , S4 都相切, 还可以作一个半径为R的球和 四面体的各棱都相切,求证:这个四面体是正四面体. 证明:设Si 的半径为ri (i = 1, 2, 3, 4),则Ai Aj = ri + rj (1 i&j 4).设O到A2 A3 A4 的投影为O1 ,由O到A2 A3 ,A3 A4 ,A4 A2 的距离相等, 得到O1 到 A2 A3 A4 的三边距离相1 (A2 A3 + A3 A4 ? 等.即O1 为 A2 A3 A4 的内心,设O到A2 A3 的投影为B ,即O1 到A2 A3 的投影. 而BA3 = 2 2 A2 A4 ) = r3 ,OB = R. 若半径为r的球与四个球均外切,则A3 O = r + r3 ,(r + r3 )2 = r3 + R 2 , r3 = 2 若半径为r的球与四个球均内切,则A3 O = r ? r3 ,(r ? r3 )2 = r3 + R 2 , r3 = r 2 ?R2 2r . R2 ?r 2 2r .类似可求得r1 , r2 , r4 均为该值,所以该四面体各条棱长相等为正四面体. 6.m个互不相同的正偶数与n个互不相同的正奇数的总和为1987, 对于所有这样的m与n,问3m + 4n的 最大值是多少?请证明你的结论. 解:设m个正偶数为a1 & a2 & ? ? ? & am ,n个正偶数为b1 & b2 & ? ? ? & bn . ∴ ai ∴ 1987 2i, bj 2j ? 1. ∵ 1987 = a1 + a2 + ? ? ? + am + b1 + b2 + ? ? ? + bn . 2 + 4 + ? ? ? + 2m + 1 + 3 + ? ? ? + 2n ? 1 = m2 + m + n2 . 5 1 1 2 设s = 3m + 4n,m = 1 3 (s ? 4n), 3 (s ? 4n)( 3 (s ? 4n) + 1) + n1987.s ? 8ns + 25n + 3s ? 12n ? 9 × 1987 s2 + (3 ? 8n)s + 25n2 ? 12n ? 9 × 19872 2 1 2 (8n 2
? n ).220. 0.1 所以判别式? = (3 ? 8n) ? 4(25n ? 12n ? 9 × 1987) = 26(1987 4 ? n2 ) & 0.s?3+61 2 ?2 2 设f (n) = 8n + 6 1987 1 ,又n为奇数. 4 ? n , f (n) = 8 ? 6n(1987 4 ? n )1不难知道n = 35时,f (n)有最大值280 + 6 所以s1 2 (280 ? + 6 762 1 4 ? 3),由s ∈ N , s762 1 4. 221.又当s = 221, n = 35, m = 27.取2, 4, . . . , 52, 60, 1, 3, . . . , 69为和为1987的35个正奇数与27个正偶数,所 以3m + 4n的最大值为221.6 第三届中国数学奥林匹克(1988年)上海 复旦大学1.设a1 , a2 , . . . , an 是给定的不全为零的实数, r1 , r2 , ? ? ? , rn 为实数,如果不等式 r1 (x1 ? a1 ) + r2 (x2 ? a2 ) + ? ? ? + rn (xn ? an ) 对任何实数x1 , x2 , ? ? ? , xn 成立,求r1 , r2 , ? ? ? , rn 的值. 解:令xi = 0(i = 1, 2, . . . , n),?(r1 a1 + r2 a2 + ? ? ? + rn an ) ∴(n i=1 n 2 2 x2 1 + x2 + ? ? ? + xn ? 2 2 a2 1 + a2 + ? ? ? + anri ai )2n2 2 ? a2 1 + a2 + ? ? ? + an .i=1 na2 i.2 2 a2 1 + a2 + ? ? ? + an .令xi = 2ai (i = 1, 2, . . . , n),r1 a1 + r2 a2 + ? ? ? + rn an ∴( ∴(i=1 n i=1ri ai )2 ri ai )2 =i=1 n i=1a2 i. a2 i.n i=1 2 ri )( n i=1 n由Cauchy不等式, (a2 i)2 ri ? n i=1(n又令xi = ri (i = 1, 2, . . . , n), 由 ∴n i=1 n i=1i=1 n i=1 2, riri ai )2 ,n i=1 n2 ri1.n i=1i=1ri ain 2 rii=12? ria2 i.ri ai =n i=1a2 i,n i=12 ri1.r1 a1i=12 ri = 1,由Cauchy不等式取等号的条件知=r2 a2= ??? =rn an .不难解得ri =ai a2 1 + a2 2 + ? ? ? + a2 n(i = 1, 2, . . . , n).2.设C1 , C2 为 同 心 圆,C2 的 半 径 是C1 的 半 径 的2倍, 四 边 形A1 A2 A3 A4 内 接 于C1 , 设A4 A1 延 长 线 交 圆C2 于B1 , A1 A2 延长线交C2 于B2 , A2 A3 延长线交圆C2 于B3 , A3 A4 延长线交圆C2 于B4 . 试证:四边形B1 B2 B3 B4 的周长 2(四边形A1 A2 A3 A4 的周长).并确定等号成立的条件. 证明:设圆心为O,连结OB1 , OB4 , OA4 ,设C1 的半径为R,C2 的半径为2R. 在四边形B4 A4 OB1 中,由Ptolemy定理,OA4 × B1 B4 + OB1 × A4 B4 R × B1 B4 + 2R × A4 B4 同理B1 B2 2R × A4 B1 ,即B1 B4 2A4 B1 ? 2A4 B4 . 2A3 B4 ? 2A3 B3 . 2A1 B2 ? 2A1 B1 ,B2 B3 2A2 B3 ? 2A2 B2 ,B3 B4 OB4 × A4 B1 .相加得B1 B2 + B2 B3 + B3 B4 + B4 B12(A1 A2 + A2 A3 + A3 A4 + A4 A1 ).即四边形B1 B2 B3 B4 的周长 2(四边形A1 A2 A3 A4 的周长). 等号成立时OAi Bi Bi+1 共圆,∠Ai+1 Ai O = ∠Bi+1 Bi O = ∠Bi Bi+1 O = ∠Ai?1 Ai O, ∴ Ai+1 Ai = Ai?1 Ai ,(i = 1, 2, 3, 4, A5 = A1 , A0 = A4 , B5 = B1 ). ∴ A1 A2 A3 A4 为菱形,又为圆内切四边形,所以A1 A2 A3 A4 为正方形. 3.在有限的实数列a1 , a2 , ? ? ? , an 中, 如果一段数ak , ak+1 , ? ? ? , ak+l?1 的算术平均值大于1988, 那么我们 把这段数叫做一条“龙”,并把ak 叫做这条龙的“龙头” (如果某一项an & 1988,那么单独这一项也叫 龙). 假设以上的数列中至少存在一条龙, 证明:这数列中全体可以作为龙头的项的算术平均数也必定大7 于1988. 证明:引理:设ak , ak+1 , . . . , ak+m?1 均可作为龙头,ak+m 不能作为龙头,或k + m ? 1 = n, 则ak , ak+1 , . . . , ak+m?1 的算术平均值大于1988. 引理的证明:对m用数学归纳法,m = 1时,设以ak 为龙头的一条龙为ak , ak+1 , . . . , ak+l?1 . 若l = 1,ak & 1988,显然成立. 否则l & 1,由ak , ak+1 , . . . , ak+l?1 算术平均值大于1988,ak+1 不是龙头, ak+1 , . . . , ak+l?1 算术平均值不 大于1988,ak & 1988,结论成立. 设小于m时结论均成立(m 1 l 2),设以ak 为龙头的一条龙为ak , ak+1 , . . . , ak+l?1 . m时,ak , ak+1 , . . . , ak+l?1 算术平均值大于1988, 由归纳假设ak+l , . . . , ak+m?1 算术平均值大于1988,结论成立. l & m时,由ak+m 不是龙头,ak+m , ak+m+1 , . . . , ak+l?1 算术平均值不大于1988, ak , ak+1 , . . . , ak+l?1 算术 平均值大于1988,结论显然也成立. 综上所述,由数学归纳法,引理成立. 设所有的龙头为ai1 , ai1 +1 , . . . , ai1 +j1 ?1 , ai2 , ai2 +1 , . . . , ai2 +j2 ?1 , . . . , aik , aik +1 , . . . , aik +jk ?1 , 其中j1 , j2 , . . . , jk 也大于1988.证毕. 4.(1)设三个正实数a, b, c满足 (a2 + b2 + c2 )2 & 2(a4 + b4 + c4 ). 求证:a, b, c一定是某个三角形的三条边长. (2)设n个正实数a1 , a2 , ? ? ? , an 满足2 2 2 4 4 4 (a2 1 + a2 + ? ? ? + an ) & (n ? 1)(a1 + a2 + ? ? ? + an )1 且im+1 & im + jm (m = 1, 2, . . . , k ? 1, k1).由引理:aim , aim +1 , . . . , aim +jm ?1 的算术平均值大于1988(m = 1, 2, . . . , k ). 所以所有龙头的算术平均值其中n3. 求证:这些数中任何三个一定是某个三角形的三条边长. a + b,则 2(a4 + b4 + c4 ) ? (a2 + b2 + c2 )2 = a4 + b4 + c4 ? 2a2 b2 ? 2b2 c2 ? 2c2 a2 = ?(a + b + c)(a + b ? c)(b + c ? a)(c + a ? b) 0证明:(1)若不然,不妨设c矛盾.∴ a, b, c为某个三角形三边长. (2)n = 3即为(1)中的情况,n & 3时,若存在某三个不是某个三角形三条边长,不妨设为a1 , a2 , a3 .则由均 值不等式4 4 2 2 2 2 (n ? 1)(a4 1 + a2 + ? ? ? + an ) & (a1 + a2 + ? ? ? + an ) 2 2 2 a2 a2 + a2 1 + a2 + a3 2 + a3 + 1 + ? ? ? + a2 n 2 2 2=(n ? 1)2 2 a2 1 + a2 + a3 22+2 2 a2 1 + a2 + a3 22+ ? ? ? + a4 n8 2 2 2 2 4 4 4 2 2 2 2 4 4 4 可得 1 2 (a + b + c ) & a + b + c ,(a + b + c ) & 2(a + b + c ).但由(1),a1 , a2 , a3 为某个三角形三边长,矛盾.所以这些数中任何三个一定是某个三角形的三条边长. 5.给出三个四面体Ai Bi Ci Di (i = 1, 2, 3), 过点Bi , Ci , Di 作平面αi , βi , γi (i = 1, 2, 3), 分别与棱Ai Bi , Ai Ci , Ai Di 垂直(i = 1, 2, 3), 如果九个平面αi , βi , γi (i = 1, 2, 3),相交于一点E , 而三点A1 , A2 , A3 在同一直 线l上, 求三个四面体的外接球面的交集(形状怎样?位置如何?) 解:∵ Ai Bi ⊥ αi 于Bi ,而E 在αi 上,∴ Ai Bi ⊥ Bi E, Bi 在以Ai E 为直径的球上.同理Ci , Di 也在以Ai E 为直 径的球上,Ai Bi Ci Di 的外接球即为在以Ai E 为直径的球. 若E 在l上,显然这三个球的中心也都在l上,它们必在E 处两两相切,交集为E . 否则E 不在l上,三个球的球心在同一条直线上( EA1 A2 中位线所在直线),且这三个球都过点E ,交集为 一个圆,直径为EE ,其中E 为E 到l的垂足. 6.如n是不小于3的自然数,以f (n)表示不是n的因子的最小自然数, 例如f (12) = 5.如果f (n) 作f (f (n)). 类似地,如果,f (f (n)) 解:设n = 2k ? m(m为奇数). 若k = 0,n为奇数,f (n) = 2, ln = 1. 若k & 0,考虑所有小于2k+1 的正奇数,若它们均为n的因子,由2k+1 n且小于2k+1 的偶数t = 2p ? q (p k, q 为奇数),由q |n, 2p |n, gcd(q, 2p ) = 1,知t|n,∴ f (n) = 2k+1 ,f (f (n)) = 3, f (f (f (n))) = 2, ln = 3. 否则取最小的t|n,t必为奇数,否则t必有一个奇因子不整除n. ∴ f (n) = t, f (f (n)) = 2, ln = 2. 综上所述, ln = ? ? ? 1, ? ? n奇数 3,又可3,又可作f (f (f (n))),等等. 如果f (f (? ? ? f (n) ? ? ? )) = 2, 共有k 个f ,就 3),求 ln .并证明你的结论.把k 叫做n的“长度”. 如果 ln 表示n的长度,试对任意自然数n(n2, n = 2k ? m(m为奇数)所有小于2k+1 的正奇数不全整除n ? ? ? ? 3, n = 2k ? m(m为奇数)所有小于2k+1 的正奇数均整除n9 第四届中国数学奥林匹克(1989年)合肥 中国科技大学1.在 半 径 为1的 圆 周 上,任 意 给 定 两 个 点 集A, B , 它 们 都 由 有 限 段 互 不 相 交 的 弧 组 成, 其 中B 的 每π 段的长度都等于 m , m是自然数. 用Aj 表示将集合A逆时针方向在圆周上转动 jπ m 弧度所得的集合(j = 1, 2, ...). 求证:存在自然数k ,使得L(Aj ∩ B )1 2π L(A)L(B ).这里L(X )表示组成点集X 的互不相交的弧的长度之和.?j 证明:我们把圆周上的点集E 沿顺时针方向在圆周上转动 jπ ,于是L(Aj ∩ B ) = m 弧度所得的集合记为EL(A ∩ B ?j ).π 设b1 , b2 , . . . , bn 为组成B 的弧段,由已知它们两两不交且每段的长度均为 m ,因此有 2m 2mL(Aj ∩ B ) =j =1 j =1 2mL(A ∩ B ?j )?j L(A ∩ (∪n i=1 bi )) j =1 2m n j L(A ∩ b? i ) j =1 i=1 n 2m j L(A ∩ b? i ) i=1 j =1 n=== =i=1m ?j L(A ∩ (∪2 j =1 bi ))因为L(bi ) = ∴2m j =1π 2m ?j m ,所以∪j =1 bi 恰好是整个圆周,从而有m ?j L(A ∩ (∪2 j =1 bi )) = L(A).L(Aj ∩ B ) = nL(A),至少存在一个k, 1 L(Aj ∩ B )k2m,使得1 n L(A) = L(A)L(B ). 2m 2π2.设x1 , x2 , ? ? ? , xn 都是正数(n2).且 x1 + x2 + ? ? ? + xn = 1.求证:n√i=1xi 1 ? xi√1 n?1n i=1√xi .证明:不妨设x1x2???xn ,则 √ 1 1 ? x1 √ 1 1 ? x2 ??? √ 1 1 ? xn由Chebyshev不等式n√i=1xi 1 ? xi1 nnnxii=1 i=1√1 1 ? xi=1 nn√i=11 1 ? xi10 由Cauchy不等式n i=1√n1 ? xii=1√1 1 ? xin2又n i=1 n√n1 ? xinni=1(1 ? xi ) = n √ 1 ? xin(n ? 1) n n(n ? 1) n n?1∴i=1√xi 1 ? xi1 n√i=11 1 ? xin i=1=而 √ 1 n?1n i=1 n√xi√1 n?1 √nni=1xi =n i=1n n?1∴i=1√xi 1 ? xi1 n?1√xi .3.设S 为 复 平 面 上 的 单 位 圆 周 (即 模 为1的 复 数 的 集 合),f 为 从S 到S 的 映 射,对 于 任 意 z ∈ S ,定 义f (1) (z ) = f (z ), f (2) (z ) = f (f (z )), ? ? ? , f (k) (z ) = f (f (k?1) (z )). 如果 c ∈ S ,使得f (1) (c) = c, f (2) (c) = c, ? ? ? , f (n?1) (c) = c, f (n) (c) = c. 则称 c 为f 的n?周期点.设m是大于1的自然数, f 定义为f (z ) = z m , 试计算f 的1989-周期点的个数. 解:记An = {z ∈ S |z 是f 的n ? 周期点},Bn = {z ∈ S |fn (z ) = z }为fn 的 不 动 点 集 合,显 然An ? Bn ,又f1 (z ) = z m ,∴ fn (z ) = z mn n∴ fn (z ) = z ? z m = z ,又|z | = 1, ∴ z m 我们证明Bn , An 有如下性质: (1)若k |n,则Bk ? Bn?1= 1, |Bn | = mn ? 1.事实上,令n = kq ,若c ∈ Bk , fk (c) = c,则fn (c) = fkq (c) = fk (fk (? ? ? fk (c) ? ? ? )) = c. ∴ c ∈ Bn , Bk ? Bn . (2)Bk ∩ Bn = Bgcd(k,n) , gcd(k, n)为k 与n的最大公约数. 由(1),Bgcd(k,n) ? Bk , Bgcd(k,n) ? Bn , ∴ Bgcd(k,n) ? Bk ∩ Bn . 反之,设c ∈ Bk ∩ Bn ,fk (c) = c, fn (c) = c,不妨设k & n. 则fn?k (c) = fn?k (fk (c)) = fn (c) = c,由辗转相 除法知fgcd(k,n) (c) = c, ∴ c ∈ Bgcd(k,n) , Bk ∩ Bn ? Bgcd(k,n) . ∴ Bk ∩ Bn = Bgcd(k,n) . (3)c ∈ Bn \ An ? ?k & n, k ∈ N? ,使k |n且c ∈ Bk . 充分性是显然的(由(1)),设c ∈ Bn \ An , fn (c) = c.且存在l & n,使得fl (c) = c,设k = gcd(l, n),则fk (c) = c, c ∈ Bk ,且k2 q个l & n, k |n.证毕.由1989 = 3 × 13 × 17,若k |1989,且k & 1989,k 必 整 除3 × 13 × 17, 32 × 13, 32 × 17中 至 少 一 个. ∴ Bk ? B663 ∪ B153 ∪ B117 , ∴ A1989 = B1989 \ (k|1989 k&1989Bk ) = B1989 \ (B663 ∪ B153 ∪ B117 ).11 由容斥原理f 的1989-周期点个数为 |A1989 | = |B1989 | ? |B663 | ? |B153 | ? |B117 | + |B663 ∩ B153 | + |B663 ∩ B117 | + |B117 ∩ B153 | ?|B663 ∩ B153 ∩ B117 | = |B1989 | ? |B663 | ? |B153 | ? |B117 | + |B51 | + |B39 | + |B9 | ? |B3 | = (m1989 ? 1) ? (m663 ? 1) ? (m153 ? 1) ? (m117 ? 1) + (m51 ? 1) + (m39 ? 1) +(m9 ? 1) ? (m3 ? 1) = m1989 ? m663 ? m153 ? m117 + m51 + m39 + m9 ? m3 4.设点D, E, F 分别在 ABC 的三边BC, CA, AB 上, 且 AEF, 径r, 又以r0 和R分别表示 DEF 和 ABC 的内切圆半径. 求证:r + r0 = R. 证明:设 ABC 周长为l,面积为S ,内切圆为 I , 在各边的切点为P, Q, R, DEF 周长为l ,面积为S . AEF, BF D, CDE 的面积分别为S1 , S2 , S3 ,内切圆分别为 I1 , I2 , I3 , BF D, CDE 的内切圆有相等的半在各边的切点为Pi , Qi , Ri (i = 1, 2, 3). 由面积公式2S = Rl, 2S = r0 l , 2S1 = r(AE + EF + F A), 2S2 = r(BD + DF + F B ), 2S3 = r(CD + DE + EC ). 又S = S + S1 + S2 + S3 , ∴ Rl = r0 l + r(l + l ),即(R ? r)l = (r + r0 )l . 又 AR1 BQ2 BP2 CP3 CR3 r AQ1 = = = = = = AQ AR BQ BP CP CR R ∴ l ? Q1 Q2 ? P2 P3 ? R1 R3 r = l R又Q1 Q2 + P2 P3 + R1 R3 = Q1 F + F Q2 + P2 D + DP3 + R3 E + ER1 = P1 F + R2 F + DR2 + DQ3 + EQ3 + EP1 = l . ∴l l=1?r R .∴(R ? r)R = (r + r0 )(R ? r), R = r + r0 .证毕.5.空间中有1989个点,其中任何三点不共线, 把它们分成点数各不相同的30组, 在任何三个不同的组中 各取一点为顶点作三角形, 求三角形个数的最大值. 解:由分组情况有限,三角形个数必存在最大值,设分为30组,各组点数为x1 & x2 & ? ? ? & x30 , 三角形个 数为f (x1 , x2 , . . . , x30 ) =1 i&j&k 30xi xj xk .12 若存在i ∈ {1, 2, . . . , 29}, xi+1 ? xi3, 则将(x1 , x2 , . . . , x30 )调整为(x1 , . . . , xi + 1, xi+1 ? 1, . . . , x30 ).f (x1 , . . . , xi + 1, xi+1 ? 1, . . . , x30 ) ? f (x1 , x2 , . . . , x30 ) = [(xi + 1 + xi+1 ? 1)1 j&k 30 j,k=i,i+1xj xk + (xi + 1)(xi+1 ? 1)j =i,i+1xj ]?[(xi + xi+1 )1 j&k 30 j,k=i,i+1xj xk + xi xi+1j =i,i+1xj ]=(xi+1 ? xi ? 1)j =i,i+1xj & 0 2, xj +1 ? xj 2, 将xi 调整为xi +1,xj +1 调f 值增大,类似的,若存在i, j ∈ {1, 2, . . . , 29}, i & j, xi+1 ? xi 整为xj +1 ? 1,f 值增大.所以当f 取最大值时,x1 , x2 , . . . , x30 中相邻两个的差最多有一个是2,其余均为1. 如果所有的均为1,1989 = x1 + (x1 + 1) + ? ? ? + (x1 + 29) = 30x1 + 435,x1 不是整数,矛盾. 设xt+1 ? xt = 2, 1 t 29, 则1989 = x1 + x2 + ? ? ? + x30 = 30x1 + (1 + 2 + ? ? ? + t ? 1) + (t + 1 + ? ? ? + 30) = 30x1 + 465 ? t. 30x1 ? t = 1524, x1 = 51, t = 6.此时各组的点的个数分别为51,52,. . . ,56,58,59,. . . ,81. 6.设f : (1, +∞) → (1, +∞)满足以下条件: 对于任意实数x, y & 1,及u, v & 0,有 f (xu y v ) 试确定所有这样的函数f .t 解:令x = y, u = v = 2 (t & 0),则f (xt )1 1f (x) 4u f (y ) 4v .(f (x)) t .1以xt 代x, 1 t 代t,则f (x) ∴ f (xt ) = (f (x)) t .1(f (xt ))t .1 1设f (e) = c, c & 1,则f (x) = f (e) ln x = c ln x . 另外,当f (x) = c ln x (c & 1)时,f (xu y v ) = c u ln x+v ln y , f (x) 4u f (y ) 4v = c 4u ln x + 4v ln y . 由Cauchy不等式,(u ln x + v ln y )( 4u 1 ln x + ∴ f (xu y v ) f (x) 4u f (y ) 4v .1 1 1 1 1 1 1 1 11 4v ln y )1.∴1 u ln x+v ln y1 4u ln x+1 4v ln y .所以所求函数为f (x) = c ln x (c & 1).13 第五届中国数学奥林匹克(1990年)郑州 《中学生数理化》编辑部1.在凸四边形ABCD中,AB 与CD不平行, O2 过C ,D且与边AB 相切于Q, 证明:分两部分证明结论. (1)EF 平分P Q的充要条件为P C ? P D = QA ? QB . 设EF 与P Q交于K ,直线P Q于 O1 , O2 分别交于J, I . ∵ P C ? P D = P I ? P Q, QA ? QB = P Q ? QJ , KQ ? KI = KE ? KF = KP ? KJ . ∴ KQ ? (KP + IP ) = KP ? (KQ + QJ ), KQ ? IP = KP ? QJ . ∴ KP = KQ ? IP = QJ ? P C ? P D = QA ? QB . (2)BC AD充要条件为P C ? P D = QA ? QB . AD ?SD SC SA SB .O1 过A,B 且与边CD相切于P , AD.O1 与 O2 相交于E ,F .求证:EF 平分线段P Q的充分必要条件是BC设AB 与DC 交于S .BC=而SP 2 = SA ? SB, SQ2 = SC ? SD. ∴ P C ? P D = QA ? QB ? (SC ? SP )(SP ? SD) = (SB ? SQ)(SQ ? SA) ? (SC + SD)SP ? SP 2 ? SC ? SD = (SB + SA)SQ ? SQ2 ? SA ? SB ? (SC + SD)SP = (SB + SA)SQ ? (SC + SD)2 ? SA ? SB = (SA + SB )2 ? SC ? SD ?SC SD SD SC SA SB SB SA++2=++2SD SC又 SD SC& 1,SA SB& 1, ∴ P C ? P D = QA ? QB ?=SA SB? BCAD.所以EF 平分线段P Q的充分必要条件是BCAD.2.设x是一个自然数,若一串自然数x0 = 1 & x1 & x2 & ? ? ? & xl = x 满足xi?1 |xi (i = 1, 2, . . . , l), 则 称{x0 , x1 , . . . , xl }为x的一条因子链. l称为该因子链的长度. L(x)与R(x)分别表示 x的最长因子链的长 度和最长因子链的条数. 对于x = 5k × 31m × 1990n ,k, m, n都是自然数,试求L(x)与R(x).αn 1 α2 解:对 于x = pα 1 p2 ? ? ? pn ,(p1 , p2 , . . . , pn 为 互 不 相 同 的 质 数,α1 , α2 , . . . , αn 为 正 整 数).i 链{x0 , x1 , . . . , xl }是最长因子链的充要条件是 xx 均为质数(i i?1x的 因 子= 1, 2, . . . , l).事实上,对于因子链{x0 , x1 , . . . , xl },若存在i, (1 链.ii = q1 q2 ,其中q1 , q2 均为大于1的正整 l),使得 xx i?1数, 则{x0 , x1 , . . . , xi?1 , q1 xi?1 , xi , . . . , xl }是长度为l + 1的因子链, 所以{x0 , x1 , ? ? ? , xl }不是最长因子i 反 之,若 xx 均 为 质 数(i = 1, 2, . . . , l), 则x = xl = i?1xl xl?1? ??? ?x2 x1? x1 (x0 = 1)为l个 质 数 的 积.所xt x2 xt?1 ?? ? ?? x1 ? x1 是t个大于1的以l = α1 + α2 + ? ? ? + αn . 而对x的任意一个因子链{x0 , x1 , . . . , xt },x = xt = 最长因子链. 14正整数之积,而x至多写成l = α1 + α2 + ? ? ? + αn 个大于1的正整数之积,所以tl.所以{x0 , x1 , ? ? ? , xl }是 L(x) = α1 + α2 + ? ? ? + αn .xl 2 每个最长因子链对应一个排列x1 , x x1 , . . . , xl?1 , l = L(x), 为α1 个p1 ,α2 个p2 ,. . . ,αn 个pn 的一个排列.∴ R(x) =(α1 +α2 +???+αn )! . α1 !α2 !???αn !当x = 5k × 31m × 1990n = 2n × 5n+k × 31m × 1990n 时, L(x) = 3n + k + m,R(x) = 3.设函数f (x)对x (1)对任何x, y(3n+k+m)! (n!)2 (n+k)!m! .0有定义,且满足条件:y 2 y2 f ( x 2 ) + x f ( 2 );0, f (x)f (y ) x x2 .(2)存在常数M & 0,当0 求证:对任意x 令x = 0,(f (0))2 0,f (x) 证明:令x = y ,(f (x))21时,|f (x)|M.2x2 f ( x 2 ).0,∴ f (0) = 0,满足结论.假设存在x & 0,使得f (x) & x2 ,用归纳法证明 f(n x ) & 22 ?2n?1 x2 (n ∈ N) n 2 kx 2 n = 0时显然成立,设n = k 时成立,f ( 2 k) & 2?2k?1 2kx .∴ f(x 2k+1)x 2 (f ( 2 k+1 (22 ?2k?1 x2 )2 k )) & = 22 ?2(k+1)?1 x2 x 2 x 2 2( 2k ) 2( 2k )n2 即n = k + 1时也成立,所以对任意n ∈ N,f ( 2x n) & 2?2n?1 2x .又n → +∞时,2n ? 2n ? 1 → +∞, 21 n → 0. ∴ ?m1 ,当n m1 时,0 &x 2n& 1,?m2 ,当nm2 时,22n?2n?1 2x & M. 0,f (x) x2 .取m = max{m1 , m2 },0 &x 2m& 1, f ( 2x m ) & M ,矛盾.所以对任意x4.设a是给定的正整数,A和B 是两个实数,试确定方程组: x2 + y 2 + z 2 = (13a)2 x2 (Ax2 + By 2 ) + y 2 (Ay 2 + Bz 2 ) + z 2 (Az 2 + Bx2 ) = 有整数解的充分必要条件(用A, B 的关系式表示,并予以证明). 解:(2) ? 若A = 若A =B 2(1) 1 (2A + B )(13a)4 4 (2)× (1)2 ,得(A ?B 4 2 )(x+ y4 + z4 ) = 1 2 (A ?B 4 2 )(13a) .B 2 ,(1)与(2)等价,不难验证x B 2 ,则= 3a, y = 4a, z = 12a为一组解. 2(x4 + y 4 + z 4 ) = (13a)4(3)∴ 2|a,设a = 2a1 ,x4 + y 4 + z 4 = 8(13a1 )4 . 若x, y, z 不全为偶数,则必为两个奇数一个偶数,x4 + y 4 + z 4 ≡ 2 (mod 4),矛盾. ∴ 2|x, 2|y, 2|z .设x = 2x1 , y = 2y1 , z = 2z1 ,则若(x, y, z, a)为(3)的解,(x1 , y1 , z1 , a1 )也为(3)的解. 类似可 依次得到(x2 , y2 , z2 , a2 )也为(3)的解,等等.但这个过程不能一直进行下去,矛盾. 15 所以方程组有整数解的充分必要条件为A =B 2.5.设X 是一个有限集合, 法则f 使得X 的每一个偶子集E (偶数个元素组成的子集)都对应一个实数f (E ), 满足条件: (1)存在一个偶子集D,使得f (D) & 1990; (2)对于X 的任意两个不相交的偶子集A, B ,有f (A ∪ B ) = f (A) + f (B ) ? 1990. 求证:存在X 的子集P, Q,满足 (1)P ∩ Q = ?,P ∪ Q = X ; (2)对P 的任何非空偶子集S ,有f (S ) & 1990; (3)对Q的任何偶子集T ,有f (T ) 1990.证明:考虑X 的所有偶子集经法则f 得到的实数最大的一个为P ,若不止一个,取元素个数最少的一个. Q = X \ P .则P ∩ Q = ?, P ∪ Q = X . 令A = B = ?,则f (?) = 1990. 对于?S ? P, S = ?,f (P ) = f (S ) + f (P \ S ) ? 1990,显然f (P \ S ) & f (P ), ∴ f (S ) & 1990. 对于?T ? Q,若T = ?,f (T ) = 1990,否则 T = ?,由f (P ∪ T ) = f (P ) + f (T ) ? 1990 ∴ P, Q满足条件.证毕. 6.凸n边形及n ? 3条在n边形内不相交的对角线组成的图形称为一个剖分图. 求证:当且仅当3|n时,存在一个剖分图是可以一笔划的图 (即可以从一个顶点出发,经过图中各线段恰一 次,最后回到出发点). 证明:因为n ? 3条在形内互不相交的对角线将凸n边形分为n ? 2个顶点均是n边形顶点的小区域, 每个 区域的内角和不小于π ,n边形的内角和为(n ? 2)π ,所以每个小区域都是三角形. 先证必要性.用归纳法容易证明可将每个三角形区域涂成黑白两色之一,使得有公共边的三角形不同 色. 假设已按照这样的要求染色,由于剖分图为可以一笔画的圈,所以由每个顶点引出的线段都是偶数 条. 从而每个顶点都是奇数个三角形的顶点,因此以原多边形外边界为一边的三角形区域有着相同的颜 色, 不妨设为黑色;另一方面,剖分图的每条对角线都是两种不同颜色三角形的公共边, 所以设黑三角形 有m1 个,白三角形有m2 个.则n = 3m1 ? 3m2 ,所以3|n. 再证充分性,设n = 3m,多边形为A1 A2 . . . A3m .连接A1 A3i , A3i A3i+2 , A3i+2 A1 (i = 1, 2, . . . , m?1)这3m? 3条对角线, 形成m ? 1个三角形,可由A1 出发,依次走过这些三角形,再走过凸多边形即可一笔画并回到 初始点.证毕. f (P ),f (T ) 1990.16 第六届中国数学奥林匹克(1991年)武汉 华中师范大学1.平面上有一凸四边形ABCD. (1).如果平面上存在一点P ,使得 ABP, 问四边形ABCD应满足什么条件? (2).满足(1)的点P ,平面上最多有几个?证明你的结论. 解:(1)(1.1)P 在ABCD内部,若A, P, C ,B, P, D分别三点共线, 显然ABCD为平行四边形,P 为对角线的交点. 若A, P, C 不 共 线,由 于 P AB , P AD等 面 积,AP 必 经 过 对 角 线BD的 中 点,同 理CP 过BD的 中 点,必 有P 为BD的中点,所以 ABD, BCD面积相等.即一条对角线平分ABCD的面积,显然也是充分条件. BCP, CDP, DAP 面积都相等,(1.2)P 在ABCD之外,不妨设P 与B, C 在AD异侧,P 必与A, B 在CD同侧,与C, D在AB 同侧. 由 P AB, P AD面积相等,P A BD,同理P D AC .设AC, BD相交于E ,AEDP 为平行四边形.SAED = SAP D = SABP + SCDP + SP BC ? SABCD = 3SAP D ? SABCD .1 ∴ SAED = 2 SABCD .这 个 条 件 也 是 充 分 条 件,若SAED =1 2 SABCD ,作 平 行 四 边 形AEDP ,显 然P B, P C 均 在AP DCB 内.∴ SABP = SAP D = SCDP = SAED , SP BC = SAP D + SABCD ? SABP ? SCDP = SAED .P 满足要求. 所以四边形ABCD有一条对角线平分面积,或者在对角线分成的四个三角形中有一个为四边形面积的 一半. (2)由(1)知,P 在形内,形外都至多有一个,又由充要条件不同时取到,P 最多有一个. 2.设I = [0, 1],G = {(x, y )|x, y ∈ I }. 求G到I 的所有映射f ,使得对任何x, y, z ∈ I 有 (1)f (f (x, y ), z ) = f (x, f (y, z )); (2)f (x, 1) = x, f (1, y ) = (3)f (zx, zy ) = z k f (x, y ).这里,k 是与x, y, z 无关的正数.x k 解:由(3),f (x, y ) = f (y ? x y , y ? 1) = y f ( y , 1)(0 & x & y ) y y f (x, y ) = f (x ? 1, x ? x ) = y k f (1, x )(0 & y & x)再由(2),f (x, y ) = y k?1 x(0 & x & y ), f (x, y ) = xk?1 y (0 & y & x) 又x = y 时,f (x, x) = xk f (1, 1) = xk . 在(1)中,取0 & x & y & z & 1,x充分小时,y k?1 x & z, x & z k?1 y . f (f (x, y ), z ) = f (y k?1 x, z ) = z k?1 y k?1 x,f (x, f (y, z )) = f (x, z k?1 y ) = x(z k?1 y )k?1 . ∴ z k?1 = z (k?1) , (k ? 1)(k ? 2) = 0, k = 1或2. k = 1时,f (x, y ) = min{x, y };k = 2时,f (x, y ) = xy .(x & 0, y & 0) 又f (x, 0) = f (x ? 1, x ? 0) = xk f (0, 1) = 0, f (0, y ) = 0, f (0, 0) = z k f (0, 0), f (0, 0) = 0. ∴ k = 1时,f (x, y ) = min{x, y };k = 2时,f (x, y ) = xy .k = 1, k = 2时,无解. 172 3.地面上有10只小鸟在啄食,其中任5只小鸟中至少有4只在一个圆上, 问有鸟最多的圆上最少有几只鸟? 解:用10个点表示10只鸟,若其中任意四点均共圆,则十个点共圆. 否则设ABCD不共圆,过其中任意不共线的三个点可作一个圆,最多有四个Si (i = 1, 2, 3, 4). 从其余六 个点中任取一点P 与ABCD构成5点组,其中必有4点共圆,必有P 落在某个圆Si 上, 有抽屉原则,另六个 点中必有两个点落在同一个圆上,这个圆上至少有5个点. 不 妨 设 为A1 , A2 , A3 , A4 , A5 在C1 上,若 存 在P, Q不 在C1 上,考 察{A1 , A2 , A3 , P, Q}, 其 中 必 有 四 点 共 圆C2 ,显然C2 = C1 ,A1 , A2 , A3 不全在C2 上,设A1 , A2 , P, Q ∈ C2 , A3 , A4 , A5 ∈ C2 . 考察{A3 , A4 , A5 , P, Q},必有四点共圆C3 ,C3 = C1 , 设A3 , A4 , P, Q ∈ C3 , A1 , A2 , A5 ∈ C3 ,C3 = C2 . 考 察{A1 , A2 , A5 , P, Q},必 有 四 点 共 圆C4 ,C4 = C1 , P, Q ∈ C4 ,A1 , A3 中 至 少 有 一 个 属 于C4 .C4 = C2 或C3 .但是这显然均构成矛盾. 所以至多有一个点不在C1 上,又因为十个点中九点共圆而另一个不在这个圆上满足题意.所以有鸟最多 的圆上最少有九只鸟. 4.求方程x2n+1 ? y 2n+1 = xyz + 22n+1 的所有满足条件 n 解:显然x & y ,且x, y 奇偶性相同,所以x ? y2n (5 + 2 + 3 )2 1 32, z5 × 22n 的正整数解组 (x, y, z, n).1 5 × 22n + 3 (1 + 22n+1 ) =2. 5 × 22 n , 3 2 n1 当x = 3, y = 1时,z = 32n ? 3 (1 + 22n+1 )为整数,又z6 × 22n . ∴ n2,n = 2,z = 70. x(x2n ? 5 × 22n )下面证不存在其他正整数解: (1)若y = 1, x & 4,y 2n+1 + 22n+1 = 22n+1 + 1, x2n+1 ? xyz = z (x2n ? z ) ∴ x2n+1 ? xyz & 4(42n ? 5 × 22n ) = 22n+2 (22n ? 5) & 22n+1 ,矛盾. (2)若y 2,由x 2, z 5 × 22n , n 2.x2n+1 ? xyz &x[(y + 2)2n ? yz ] x[y 2n + 4ny 2n?1 + 4n(2n ? 1)y 2n?2 + 22n ? yz ] xy 2n + x ? 22n + x[4ny 2n?1 + 4n(2n ? 1)y 2n?2 ? 5 × 22n y ] & y 2n+1 + 22n+1 + xy [4ny 2n?2 + 4n(2n ? 1)y 2n?3 ? 5 × 22n ] 8 × 22n?2 + 8 × 3 × 22n?3 & 5 × 22n .∵y2,4ny 2n?2 + 4n(2n ? 1)y 2n?3∴ x2n+1 ? xyz & y 2n+1 + 22n+1 ,矛盾,所以只有一组正整数解(x, y, z, n) = (3, 1, 70, 2). 5.求所有自然数n,使得k∈Nmin (k 2 + ?n ) = 1991. k2n 2 k0这里[x]表示x的整数部分. 解:条件等价于对于?k ∈ N? , k 2 + 即对于?k ∈ N , k ? 1991k + n 即(k 2 ? ) 2 ? 4 2 1991 42n k22 1991, 且?k0 ∈ N? , k0 +& 1992.0+n?0 = 32,324 ? 1991 × 322 + n 18 0, n 1024 × 967 = 990208.取k ,使得|k ?1991 2 |最小,k 4 2 并且存在k0 ∈ N? , k0 ? 1991k0 +n&0即(k 2 ? 996)2 + n ? 9962 & 02 由|k0 ? 996|的最小值为28,所以n ? 9962 & ?282 , n & 9962 ? 282 = 1024 × 968 = 991232.∴ 990208n991231.6.MO牌足球由若干多边形皮块用三种不同颜色的丝线缝制而成, 它有以下特点: (1)任一多边形皮块的一条边恰与另一多边形皮块同样长的一条边用一种颜色的丝线缝合; (2)足球上每一个结点,恰好是三个多边形的顶点,每一结点的三条缝线颜色互不相同. 求证:可以在MO牌足球的每一结点上放置一个不等于1的复数, 使得每一多边形的所有顶点上放置的复 数的乘积都等于1. 证明:设这三种颜色为红,黄,蓝,对每条边赋值,红色为1,黄色为e 3 πi ,蓝色为e 3 πi . 对于每个节点,若三种颜色的线依逆时针方向依次为红,黄,蓝,则在这个结点放上e 3 πi , 否则放上e 3 πi .则 结点上放置的数为逆时针方向一条边的复数除以下一条边的复数, (e 3 πi =2 2 4 2 41 e 3 πi4=e 3 πi e 3 πi24=1 e 3 πi 4 πi e 3 πi e 3 πi , e 3 = 2 πi = 4 πi = ) 1 1 e3 e3224所以对于任意一个多边形,沿顺时针方向走过每条边依次为z1 , z2 , . . . , zk , 则顶点上依次放置ω1 =z1 z2 , ω2=z2 z3 , . . . , ωk=zk z1 ,∴ ω1 ω2 ? ? ? ωk = 1.满足题意,证毕.19 第七届中国数学奥林匹克(1992年)北京 北京数学奥林匹克发展中心1.设方程xn + an?1 xn?1 + an?2 xn?2 + ? ? ? + a1 x + a0 = 0的系数都是实数, 且适合条件0 & a0 a1 a2 ??? an?1 1.已知λ为方程的复根且|λ| 1. 求证:λn+1 = 1. 证明:由λn + an?1 λn?1 + an?2 λn?2 + ? ? ? + a1 λ + a0 = 0,a0 & 0, λ = 0.1 n 1 n?1 1 ∴ a0 ( λ ) + a1 ( λ ) + ? ? ? + an?1 ( λ )+1=0 1 n+1 1 n 1 2 a0 ( λ ) + a1 ( λ ) + ? ? ? + an?1 ( λ ) + 1 由|λ| & 1, | λ | 1 λ=01. ∴1 1 1 1 = a0 ( )n+1 + (a1 ? a0 )( )n + ? ? ? + (1 ? an?1 )( ) λ λ λ 1 1 1 = |a0 ( )n+1 + (a1 ? a0 )( )n + ? ? ? + (1 ? an?1 )( )| λ λ λ 1 1 1 a0 | |n+1 + (a1 ? a0 )| |n + ? ? ? + (1 ? an?1 )| | λ λ λ a0 + (a1 ? a0 ) + ? ? ? + (1 ? an?1 ) = 11?an?1 a0 a1 ?a0 取等号条件为 λn 的辐角相同, 且|λ| = 1. +1 , λn , . . . , λ a0 但是 λn +1 + a1 ?a0 λn+ ??? +1?an?1 λ= 1 所以它们均为非负实数,又a0 & 0, ∴ λn+1 ∈ R+ .又因为|λn+1 | = 1, ∴ λn+1 = 1. 2.设x1 , x2 , ? ? ? , xn 为非负实数,记xn+1 = x1 , a = min{x1 , x2 , ? ? ? , xn },试证:n i=11 + xi 1 + xi+11 n+ (1 + a)2n(xi ? a)2i=1其中等号成立当且仅当x1 = x2 = ? ? ? = xn . 证明:对n用数学归纳法,n = 1时,x1 = a,结论显然成立. 设n = k 时结论成立,当n = k + 1时,不妨设x1 = a,由归纳假设k i=11 + xi 1 + xk + 1 + xi+1 1 + x1k+1 (1 + a)2k(xj ? a)2j =1因此要证明k+1 i=11 + xi 1 + xi+1k+1+1 (1 + a)2k+1(xi ? a)2i=1只需证明 1 + xk 1 + xk+1 1 + xk + ? 1 + xk+1 1 + x1 1 + x1 即 (xk+1 ? x1 )(xk+1 ? xk ) (1 + xk+1 )(1 + x1 ) 20 (xk+1 ? a)2 (1 + a)2 (1) 1+ (xk+1 ? a)2 (1 + a)2 由xk+1x1 = a, xkx1 = a,所以(1 + xk+1 )(1 + x1 )(1 + a)2 ,(xk+1 ? x1 )(xk+1 ? xk )(xk+1 ? a)2 .(1)显然成立,等号成立时当且仅当x1 = x2 = ? ? ? = xk ,且xk+1 = xk = x1 . 所以n = k + 1时结论成立.由 归纳法,结论成立. 3.在平面上给出一个9 × 9的方格表, 并在其中每一方格中都任意填入+1或-1. 下面一种改变填入数字 的方式称为一次变动: 对于任意一个小方格,将与此格有一条公共边的所有小方格 (不包含此格本身)中 的数作连乘积,于是每取一个格, 就算出一个数.在所有小格都取遍后, 就将原来格中的数全部擦去,而 将这些算出的数填入相应的小方格中. 试问是否总可以经过有限次变动,使得所有小方格中的数都变 为+1? 解:未必,例如如下的4 × 4表格经变动后保持不变, 将它对称填入9 × 9格子的四个角的4 × 4方格, 并将 正中一行与正中一列均填上+1,该9 × 9方格表经变动后保持不变. +1 -1 -1 -1 -1 +1 -1 +1 -1 -1 +1 +1 -1 +1 +1 +14.凸 四 边 形 内 接 于 O,对 角 线AC 与BD相 交 于P , ABP 与 CDP 的 外 接 圆 相 交 于P 和 另 一 点Q, 且O, P, Q三点两两不重合.试证∠OQP = 90? . 证明:不妨设Q在∠BP C 内,连结AO, AQ, DO, DQ. 则∠AQD = ∠AQP + ∠DQP = ∠ABP + ∠DCP = 2∠ABD = ∠AOD. 所以A, D, Q, O四点共圆.1 ? ? ∴ ∠OQP = ∠AQO + ∠P QA = ∠ADO + ∠ABD = 1 2 (180 ? ∠AOD ) + 2 ∠AOD = 90 ,证毕.5.在有8个顶点的简单图中,没有四边形的图的边数的最大值是多少? (简单图是指任意顶点与自己没有 边相连,而且任意两个顶点之间至多有一条边相连的图) 解:最 大 值 为11,首 先 构 造 一 个8顶 点11条 边 的 图,其 中 没 有 四 边 形.顶 点 为Ai (i = 1, 2, . . . , 8), 边 为A1 A2 , A2 A3 , A3 A4 , A4 A5 , A5 A1 , A5 A6 , A6 A7 , A7 A8 , A8 A4 , A1 A6 , A3 A8 .不难验证这个图满足题意. 下面证明:若简单图G中有8个顶点,12条边,其中必存在四边形.若不然,设G中度数最大的顶点(中的一 个)为A,A的度数为d. 显然8d (1)若d 2 × 12 = 24,即d 3.5,设与A有边相连的顶点组成的集合为S ,|S | = d. 则S 中不会有顶点与其他两个同在S 中的顶7?d 2点相连,顶点均在S 中的边至多有[ d 2 ]条, 而其他的顶点每个至多与S 中一个顶点相连,所以图G中最多有 边f (d) = d + [ d 2] + 7 ? d + 条, 不难验证d 5时,f (d) & 12,矛盾. (2)d = 4,(1)中的讨论仍适用,此时f (d) = 12,必有所有取等号的条件都取到. 设S = {B1 , B2 , B3 , B4 },另 三个点为C1 , C2 , C3 .不妨设B1 , B2 相连,B3 , B4 相连,C1 C2 C3 构成三角形. S1 = {B1 , B2 }, S2 = {B3 , B4 }, 由于C1 , C2 , C3 每个向S 中的一个点连出一条边,必有两个同向S1 或S2 中的点连出边, 不妨设为C1 , C2 都 向S1 中 的 点 连 出 边,C1 与B1 相 连. 连,则B1 C1 C2 B2 为四边形.矛盾. 但 是 此 时,若C2 与B1 相 连,则B1 C1 C3 C2 为 四 边 形; 若C2 与B2 相21 (3)d = 3,则所有的顶点度数均为3,不妨设A,B 之间没有边相连,从它们连出的边为AAi , BBi (i = 1, 2, 3). 则S1 = {A1 , A2 , A3 }, S2 = {B1 , B2 , B3 }至多有一个公共元素. (3.1)若它们没有公共元素,A, A1 , A2 , A3 , B, B1 , B2 , B3 为全部8个点,由(1)中的讨论知,S1 ,S2 中顶点相连 各自至多有一条边. 它们之间最多有3条边,最多有11条边,矛盾. (3.2)若 它 们 有 公 共 元 素,设A3 = B3 ,第8个 点 为C ,从 它 出 发 只 能 各 向S1 ,S2 连 出 一 条 边, 而 它 又 不 与A, B 相连,所以C 的度数小于3,矛盾. 综上所述,若简单图G中有8个顶点,12条边,其中必存在四边形.所以在有8个顶点的简单图中,没有四边 形的图的边数的最大值是11. 6.已知整数序列{a0 , a1 , a2 , . . .}满足条件: (1)an+1 = 3an ? 3an?1 + an?2 , n = 2, 3, . . . (2)2a1 = a0 + a2 ? 2. (3)对任意的自然数m,存在k ,使得ak , ak+1 , . . . , ak+m?1 都为完全平方数. 试证:序列{a0 , a1 , a2 , . . .}的所有项都是完全平方数. 证明:由(1)的特征方程x3 = 3x2 ? 3x + 1的三个根均为1知an = an2 + bn + c(a, b, c为待定实数.b 2 代入(2)得a = 1,an = n2 + bn + c = (n + 2 ) +c? b2 4 .b = a1 ? a0 ? 1, c = a0 均为整数,令 (n + 只需 n& 令 n0 = [max{ 则 当n (n +b2 4b?1 2 b b2 b+1 2 ) & an = (n + )2 + c ? & (n + ) 2 2 4 2 (b ? 1)2 (b + 1)2 ? c, n & c ? . 4 4 (b + 1)2 (b ? 1)2 ? c, c ? }] + 1 4 4(?)n0 时,(?)成 立.在(3)中 令m = n0 + 1,知 道 必 存 在nb 2 = 0,∴ ?n ∈ N, an = (n + 2 ) 为完全平方数,证毕.n0 使an 为 完 全 平 方 数,必 有an =b 2 2 ) ,b为偶数.所以c ?22 第八届中国数学奥林匹克(1993年)济南 山东大学1.设n是奇数,试证明存在2n个整数a1 , a2 , . . . , b1 , b2 , . . . , bn , 使得对于任意一个整数k, 0 & k & n.下 列3n个数ai + ai+k , ai + bi , bi + bi+k (i = 1, 2, ? ? ? , n, 其中an+j = aj , bn+j = bj )被3n除时余数互不相同. 证明:令ai = 3i, bi = 3i + 1(i = 1, 2, . . . , n) 则ai + ai+k = 3(2i + k ) ≡ 0 (mod 3) ai + bi = 6i + 1 ≡ 1 (mod 3) bi + bi+k = 3(2i + k ) + 2 ≡ 2 (mod 3) 显然不同组的两数被3n除余数互不相同,只需说明同一组中任一两个数模3n互不相同, 即ci = 6i模3n互 不相同.(i = 1, 2, . . . , n) 若ci ≡ cj (mod 3n),则6(j ? i) ≡ 0 (mod 3),即n|2(j ? i),但是n为奇数,∴ n|j ? i,又因为|j ? i| & n,必 有j = i. 所以这3n个数被3n除时余数互不相同. 2.给定自然数k 及实数a & 0, 已知k1 + k2 + ? ? ? + kr = k, ki ∈ N? (i = 1, 2, . . . , r, 1 ak2 + ? ? ? + akr 的最大值. 解:当a = 1时,ak1 + ak2 + ? ? ? + akr = r,显然最大值为k . 当a & 0,且a = 1时,f (x) = ax 在[1, +∞)上为下凸函数, 即aki + akj 于它等价于a(aki ?1 ? 1)(akj ?1 ? 1) 0,两括号内显然同号. a + aki +kj ?1 (ki , kj ∈ N? . 这是由 r k ).求ak1 +所以我们可以经过r ? 1次调整将其调整为k1 = k2 = ? ? ? = kr?1 = 1, kr = k + 1 ? r的情况,而值始终不 减. 设此时的值为F (r) = (r ? 1)a + ak+1?r . 又a + am 若k + 1 ? r am+1 ? ma loga ( a? 1 ). a loga ( a? 1 ),则F (r) = (r ? 2)a + a + ak+1?ra 若k + 1 ? r & loga ( a? 1 ),则(r ? 2)a + ak+2?r(r ? 3)a + ak+3?r???ak = F (1)F (r) = ra ? a + ak+1?r 所以F 的最大值为max{ak , ka} = max{ak , ka}. 所以ak1 + ak2 + ? ? ? + akr 的最大值为 ? 1 ? ka, a k k? 1 (k 2) max{ak , ka} = 1 ? ak , k = 1或a & k k? 1 (kra + ak?r???ka = F (k )当r = k, k1 = k2 = ? ? ? = kr = 1取等号 2) 当r = 1, k1 = k 取等号3.设圆K 和K1 同心,它们的半径分别为R和R1 , R1 & R. 四边形ABCD内接于圆K ,四边形A1 B1 C1 D1 内 接于圆K1 , 点A1 , B1 , C1 , D1 分别在射线CD, DA, AB, BC 上, 求证: S A 1 B 1 C 1 D1 SABCD 232 R1 . 2 R 证明:设AB = a, BC = b, CD = c, DA = d, AB1 = w, BC1 = x, CD1 = y, DA1 = z .则 S A1 B 1 C 1 D1 SAB1 C1 SBC1 D1 SCD1 A1 SDA1 B1 w(a + x) x(b + y ) y (c + z ) z (d + w) = 1+ + + + = 1+ + + + SABCD SABCD SABCD SABCD SABCD ad + bc ab + cd ad + bc ab + cd2 由切割线定理,(a + x)x = (b + y )y = (c + z )z = (d + w)w = R1 ? R2 .要证明 2 S A1 B1 C 1 D1 R1 . 2 SABCD R w x y z + + + x(ad + bc) y (ab + cd) z (ad + bc) w(ab + cd)只需证明 即2 (R1 ? R2 )2 R1 ? R2 R2w x y z + + + x(ad + bc) y (ab + cd) z (ad + bc) w(ab + cd) √4 . (ad+bc)(ab+cd)1 R2(?)由均值不等式知(?)左边 再由均值不等式(ad + bc)(ab + cd) 1 4ad + bc + ab + cd 2 a+c+b+d 2 √ 4 2R .2=2 21 [(a + c)(b + d)]2 4 1 (a + b + c + d)4 64=由圆内接四边形中正方形周长最长知a + b + c + d ∴ (ab + cd)(bc + ad) 16R4 ,显然有(?)成立.证毕.4.给定集合S = {z1 , z2 , ? ? ? , z1993 }, 其中z1 , z2 , ? ? ? , z1993 是非零复数(可看作平面上的非零向量). 求 证:可以把S 中的元素分成若干组,使得 (1)S 中每个元素属于且仅属于其中一组; (2)每一组中任一复数与该组所有复数之和的夹角不超过90? ; (3)将任意两组中复数分别求和,所得和数之间的夹角大于90? . 证明:取S 中某些元素组成子集A,使得这些元素之和的模长最大. 由于元素个数有限,所以子集的取法有限,必然存在这样的A. 在S \ A中同样取元素之和模长最大的一些元素组成B . C = S \ A \ B ,下面我们证明A, B, C 满足题意. 若不然,设A, B, C 中元素之和分别为a, b, c, (1)若存在z ∈ A,与a夹角大于90? ,则?z 与a夹角为锐角,则 |(?z ) + a| & |a|,与A的选取矛盾.∴ A中的元 素与a的夹角均不超过90? ,类似的这对B 也成立. (2)对于z ∈ A,若z 与a的夹角不大于90? ,则 |z + a| & |a|,与A的选取矛盾. 所以任意不在A中的元素或它 们的和与a的夹角大于90? .类似的, 任意在C 中的元素或它们的和与b的夹角大于90? . 所以a, b, c两两夹角大于90? . (3)若存在z ∈ C ,z 与c的夹角大于90? ,则a, b, c, z 两两夹角大于90? ,矛盾. 所以C 中的元素与c的夹角均不 超过90? . 综上所述,A, B, C 满足题意,证毕. 24 5.10人到书店买书,已知 (1)每人都买了三种书; (2)任何两人所买的书,都至少有一种相同. 问购买人数最多的一种书最(至)少有几人购买?说明理由. 解:至少有5个人. 设A买的书为1,2,3,其余9人每人至少买这三者之一,有抽屉原则至少有4人买了同一种书,再加上A,至少 有5个人. 另外,若10个人买的书分别为(123),(123),(145),(167),(246),(246),(257),(347),(356),(356). 则不难验证 满足条件,且购买人数最多的一种书恰有5人购买. 6.设函数f : (0, +∞) → (0, +∞)满足以下条件: 对于任意正实数x, y ,有f (xy ) 试证:对任意的正实数x及自然数n,有 f (xn )1 1f (x)f (y ).f (x)f (x2 ) 2 ? ? ? f (xn ) n111n n n (x) = Fn 证明:设Fn (x) = f (x)f (x2 ) 2 ? ? ? f (xn ) n ,则Fn (x) = Fn?1 (x)f (xn ) n , Fn ?1 (x)f (x ). n?1 n?1 n?1 2 2 类似的,Fn ), ? ? ? , F2 (x) = F1 (x)f (x2 ), F1 (x) = f (x). ?1 (x) = Fn?2 (x)f (x n (x) = Fn?1 (x)Fn?2 (x) ? ? ? F2 (x)F1 (x)f (xn )f (xn?1 ) ? ? ? f (x). 相乘得Fn用归纳法证明Fn (x)f (xn ).对于n = 1显然成立,设对nk 均成立,由归纳假设k+1 k+1 Fk )f (xk ) ? ? ? f (x) +1 (x) = Fk (x)Fk?1 (x) ? ? ? F1 (x)f (xf (xk )f (xk?1 ) ? ? ? f (x)f (xk+1 )f (xk ) ? ? ? f (x) = f (xk+1 )(f (xk )f (x))(f (xk?1 )f (x2 )) ? ? ? (f (x)f (xk )) (f (xk+1 ))k+1 ∴ Fk+1 (x) ∴ Fn (x) f (xk+1 ),由数学归纳法知结论对于?n ∈ N? 均成立. f (xn ),即f (xn ) f (x)f (x2 ) 2 ? ? ? f (xn ) n .证毕.1 125 第九届中国数学奥林匹克(1994年)上海 复旦大学1.设ABCD是一个梯形(AB CD),E 是线段AB 上一点,F 是线段CD上一点, 线段CE 与BF 相交于1 4 SABCD .点H ,线段ED与AF 相交于点G, 求证:SEHF G 证明:引理:在梯形ABCD中,AC, BD交于E.(AB 引理的证明:显然SACD如果ABCD是一个任意的凸四边形, 同样结论是否成立?请说明理由. CD)则SAED AED=SBEC1 4 SABCD .=SBCD ,都减去SCDE ,即有S=SBEC ,设为S .则S S ∴SABE S CDE ABE=DE S CDE = BE S= S 2 ,由均值不等式ABESABCD = S 所以SAED+SCDE+ 2S2 SABE?SCDE+ 2S = 4S=SBEC EGF1 4 SABCD . 1 4 SAEDF , S EHF 1 4 SBECF .回到原题,由引理,S 相加得SEHF G1 4 SABCD .如果ABCD是一个任意的凸四边形,结论未必成立. 当DA → 0, E → B, F → C 时,SEF GH → SABCD .BE CF 1 所以当 AD BC , AB , CD 足够小时,SEF GH & 4 SABCD .2.n(n4)个盘子里放有总数不少于4的糖块, 从任意的两个盘子各取一块糖,放入另一个盘子中, 称为一次操作,问能否经过有限次操作, 将所有的糖块集中到一个盘子中去?证明你的结论. 解:能够做到.用数学归纳法证明,设有m块糖,(m 4).当m = 4时,至多有4个盘子中有糖,只有下面几种情况,不难看出结论均成立. (1)(1, 1, 1, 1) → (3, 1, 0, 0) → (2, 0, 2, 0) → (1, 0, 1, 2) → (0, 0, 0, 4) (2)(2, 1, 1, 0) → (4, 0, 0, 0) (3)(3, 1, 0, 0) → (2, 0, 2, 0) → (1, 0, 1, 2) → (0, 0, 0, 4) (4)(2, 2, 0, 0) → (1, 1, 2, 0) → (0, 0, 4, 0) (5)(4, 0, 0, 0) 设当小于m时结论均成立(m & 4),当有m块糖时,可以先将m ? 1块糖集中到一个盘子内, 由归纳假设这 是可以做到的.剩下的一块若也在同一个盘子内,显然结论成立.否则可由如下的过程将m块糖集中到一 个盘子内. (m ? 1, 1, 0, 0) → (m ? 2, 0, 2, 0) → (m ? 3, 2, 1, 0) → (m ? 4, 1, 1, 2) → (m ? 2, 0, 1, 1) → (m, 0, 0, 0) 由归纳法,总可经有限次操作将所有的糖集中到同一个盘子中. 3.求适合以下条件的所有函数f : [1, +∞) → [1, +∞), (1)f (x) 2(x + 1); 26 (2)f (x + 1) =(f (x))2 ?1 . x解:显然f (x) = x + 1是一个符合条件的函数,若存在f 满足条件,且?x0 ∈ [1, +∞),使得f (x0 ) = x0 + 1. (1)f (x0 ) & x0 + 1,设g (x) = f (x) ? x ? 1,g (x0 ) & 0,又由于f (x)1 而且由条件(2)不难得到g (x + 1) = 2g (x) + x [(g (x))2 + 2g (x)].2(x + 1),得到g (x)x + 1.∴ g (x0 + 1) & 2g (x0 ) & 0,由数学归纳法不难证明g (x0 + n) & 2n g (x0 ),当n充分大时,g (x0 + n) & 2n g (x0 ) & x0 + n + 1,矛盾. (2)f (x0 ) & x0 + 1,设g (x) = x + 1 ? f (x),g (x0 ) & 0,由f (x) ∴ g (x0 + 1) 1,得到g (x) x.1 而且由条件(2)不难得到g (x + 1) = 2g (x) ? x [(g (x))2 ? 2g (x)].2g (x0 ) ? (g (x0 ) ? 2) = g (x0 ) + 2,由数学归纳法不难证明f (x0 + n) g (x0 ) + 2n & x0 + n,矛盾.g (x0 ) + 2n,当n充分大时,g (x0 + n)综上所述,对于任意的x ∈ [1, +∞),f (x) = x + 1,所以f (x) = x + 1是唯一满足条件的函数. 4.已知f (z ) = C0 z n + C1 z n?1 + C2 z n?2 + ? ? ? + Cn?1 z + Cn 是一个n次复系数多项式, 求证:一定存在 一个复数z0 , |z0 | 1,满足|f (z0 )| |C0 | + |Cn |.2π i n证明:取ω ,|ω | = 1,使得C0 ω n 与Cn 辐角相同,(Cn = 0时,ω = 1即可). ε = e 令zi = ωεi?1 (i = 1, 2, . . . , n),则为n次单位根.f (z1 ) + f (z2 ) + ? ? ? + f (zn ) = n(C0 ω n + Cn ), |z1 | = |z2 | = ? ? ? = |zn | = 1n∴i=1|f (zi )||f (z1 ) + f (z2 ) + ? ? ? + f (zn )| = n|C0 ω n + Cn | = n(|C0 | + |Cn |) |C0 | + |Cn |, |zi | = 1,显然结论成立.n k=0所以必然存在i,使得f (zi )5.对任何自然数n,求证恒等式:n k n?k 2 k k [ n? 2 ]2n+1 n n=2n + 1 n其中0 0=k n?k 1 , [ n? 2 ]表示 2 的整数部分.证明:考虑函数f (x) = (1 + x)2n+1 ,显然 另一方面2为它的n次项系数.nf (x) = (x + 2x + 1) (x + 1) =k=1(x + 1)(x2 + 1)n?k (2x)kn k n?kn?k 2考虑每一项中xn 的系数,若n ? k 为偶数,(x + 1)(x2 + 1)n?k 中xn?k 的系数为 数,(x + 1)(x2 + 1)n?k 中xn?k 的系数为 所以每一项中xn 的系数均为 f (x)中xn 的系数为 n?kn?k?1 2; 若n ? k 为奇.n k n?k 2 . k k [ n? 2 ]n k=0 nn k n?k 2 k k [ n? 2 ] = 2n + 1 n∴k=0n k n?k 2 k k [ n? 2 ] 27 6.设M 为平面上坐标为(1994p, 7 × 1994p)的点, 其中p是素数,求满足下述条件的直角三角形的个数: (1)三角形的三个顶点都是整点,而且M 是其直角顶点; (2)三角形的内心是坐标原点. 解:设该直角三角形为M AB ,并且M A斜率为正. 将坐标原点平移至M ,设M A, M O的倾斜角分别为α, β ,则tan β = 7. 所以M A的斜率为k = tan α = tan(β ? π 4) =tan β ?1 1+tan β=3 4.4 M B 的斜率为? 3 ,设A(?4t, ?3t), B (3t , ?4t ),由A, B 为整点,t, t 为正整数,M A = 5t, M B = 5t . √ 由内心性质,并且∠M 是直角,M A + M B ? AB = 2r, 2r = M O = (1994p)2 + (7 × 1994p)2 ,M A & r, M B & r.(r为内切圆半径) √ ∴ 5t + 5t ? 5 t2 + t 2 = 1 × 5 × 1994p, t2 + t 2 = (t + t ? 2 × 1994p)2 , tt ? 2 × 1994p(t + t ) + 2 × 19942 p2 = 0(t & 1994p, t & 1994p) (t ? 2 × 1994p)(t ? 2 × 1994p) = 2 × (1994p)2 设m = t ? 2 × 1994p, n = t ? 2 × 1994p.mn = 23 × 9972 × p2 . 不难知道m, n均为正整数,所以(m, n)一组正整数解对应一个直角三角形. ∴ 直角三角形的个数为23 × 9972 × p2 的正因子个数. p = 2, p = 997时为(3 + 1)(2 + 1)(2 + 1) = 36; p = 2时为(5 + 1)(2 + 1) = 18; p = 997时为(3 + 1)(4 + 1) = 20.28 第十届中国数学奥林匹克(1995年)合肥 中国科技大学1.设2n个实数a1 , a2 , . . . , b1 , b2 , . . . , bn (n (1)a1 + a2 + ? ? ? + an = b1 + b2 + ? ? ? + (2)0 & a1 = a2 , ai + ai+1 = ai+2 (i = 1, 2, . . . , n ? 2); (3)0 & b1 证明:若a1 若存在2 i b2 , bi + bi+1 bi+2 (i = 1, 2, . . . , n ? 2). bi (i = 1, 2, . . . , n),显然结论成立. bj (j i),所以也有结论成立. 求证:an?1 + an bn?1 + bn . bi , ai+1 bi+1 . 3)满足b1 ,则由递推关系不难证明ai n,使得aii = n ? 1时,显然有结论成立;i & n ? 1时,由递推关系不难证明aj 否则必有a1 & b1 ,设I = {i|ai 若i ∈ I ,则ai bi },则I 中不存在相邻的正整数. n ? 2). ai &i∈I ∪I i∈I ∪I设I = {j |j = i ? 1, i ∈ I }, I ∩ I = ?. bi , ai?1 & bi?1 , ai+1 & bi+1 (i bi + bi?1 . ∴n i=1∴ ai + ai?1 = ai+1 & bi+1 又若i ∈ I ∪ I ,ai & bi . ∴n?2 i=1 n?2 i=1 nbi (in ? 2).ai &bi , 又i=1ai =bi , ∴ an?1 + anbn?1 + bn .综上所述,an?1 + anbn?1 + bn .2.设N为自然数集合, f : N → N适合条件:f (1) = 1,对于任何自然数n都有 ? 3f (n)f (2n + 1) = f (2n)(1 + 3f (n)); ? f (2n) & 6f (n). 试求方程f (k ) + f (l) = 293,其中k & l的所有解. 解:由gcd(3f (n), 1 + 3f (n)) = 1, ∴ 3f (n)|f (2n). 又因为f (2n) & 6f (n),所以f (2n) = 3f (n), f (2n + 1) = 3f (n) + 1 = f (2n) + 1. 由f (1) = 1,由归纳法不难证明:若n的二进制表示为 (am am?1 . . . a0 )2 = am 2m + am?1 2m?1 + ? ? ? + a0 (ai = 0或1) 则f (n)的三进制表示为 (am am?1 . . . a0 )3 = am 3m + am?1 3m?1 + ? ? ? + a0 (ai = 0或1) 由k & l,显然f (k ) & f (l), 设k = (am am?1 . . . a0 )2 , l = (bm bm?1 . . . b0 )2 , ci = ai + bi , ci = 0, 1, 2(i = 0, 1, . . . , m). 则f (k ) + f (l) = (cm cm?1 . . . c0 )3 . 而293 = 35 + 33 + 2 × 32 + 3 + 2 = ( .所以由f (k ) & f (l), am 不难知道只有四组解: 29 bm , m = 5. 必有a5 = 0, b5 = 1, a4 = b4 = 0, a2 = b2 = a0 = b0 = 1.a3 + b3 = 1, a1 + b1 = 1. (1)a3 = a1 = 1,f (k ) = (1111)3 , f (l) = ( , (k, l) = (15, 37); (2)a3 = 0, a1 = 1,f (k ) = (111)3 , f (l) = ( , (k, l) = (7, 45); (3)a3 = 1, a1 = 0,f (k ) = (1101)3 , f (l) = ( , (k, l) = (13, 39); (4)a3 = a1 = 0,f (k ) = (101)3 , f (l) = ( , (k, l) = (5, 47). 3.试求10 10 10|k (x + y ? 10i)(3x ? 6y ? 36j )(19x + 95y ? 95k )|i=1 j =1 k=1的最小值,其中x和y 是任意实数. 解:设10 10 10F =i=1 j =1 k=1 10|k (x + y ? 10i)(3x ? 6y ? 36j )(19x + 95y ? 95k )|10 10F1 =i=1|x + y ? 10i|, F2 =j =1|x ? 2y ? 12j |, F3 =k=1k |x + 5y ? 5k |则F = 57F1 F2 F3 . 引理:a1 , a2 , . . . , an 为n个实数,a1 若2|n, n = 2m, am b am+1 ; 若2 n, n = 2m + 1, b = am+1 . 设g (x) = |x ? a1 | + |x ? a2 | + ? ? ? + |x ? an |,则g (t)的最小值为g (b). 引理的证明:∵ |x ? a1 | + |x ? an | |x ? a2 | + |x ? an?1 | n为奇数时,|x ? a n+1 |2a2???ak .定义b为它们的中位数:an ? a1 (当a1 xxan 时取等号) x 时取等号) an 2 +1an?1 ? a2 (当a2an?1 时取等号)? ? ?n为偶数时,|x ? a n | | + |x ? a n 2 2 +1 ∴ g (x)an ? an (当a n 2 +1 2 220(当x = a n+1 时取等号)2(an + an?1 + ? ? ? + a[ n ) ? (a[ n+1 ] + ? ? ? + a1 ) (当x = b时取等号) 2 ]+1 x+y x ? 2y 60时取等号. 72时取等号.回到原题,由引理分别应用到F1 , F2 , F3 上得 F1 F2 F3 ∴F 10 × (10 + 9 + 8 + 7 + 6 ? 5 ? 4 ? 3 ? 2 ? 1) = 250, 50 12 × (10 + 9 + 8 + 7 + 6 ? 5 ? 4 ? 3 ? 2 ? 1) = 300, 605 × (10 × 10 + 9 × 9 + 8 × 8 ? 7 × 6 ? 6 × 6 ? 5 × 5 ? 4 × 4 ? 3 × 3 ? 2 × 2 ? 1) = 560, x + 5y = 35时取等号. 57 × 250 × 300 × 560 = .且不难验证x = 60, y = ?5时满足所有取等号条件,所以原式的最小值为. 4.空间有四个球,它们的半径分别为2,2,3,3,每个球都与其余3个球外切, 另有一个小球与这四个球都外 切,求该小球的半径. 解:设四个球的球心分别为A, B, C, D,则AB = 4, CD = 6, AC = BC = AD = BD = 5. 设E, F 分别为AB ,CD中点,小球球心为O,半径为r,则四面体ABCD关于平面ABF ,CDE 对称. 四个球也同样,所以由对称性O在EF 上. √ √ OE = OA2 ? AE 2 = (2 + r)2 ? 22 = r2 + 4r,30 √ OD2 ? DF 2 = (3 + r)2 ? 32 = r2 + 6r, √ √ √ √ EF = F A2 ? AE 2 = DA2 ? DF 2 ? AE 2 = 52 ? 32 ? 22 = 2 3. √ √ √ ∴ r2 + 4r + r2 + 6r = 2 3. OF = 解得r =6 11 .√5.设a1 , a2 , . . . , a10 是任意10个两两不同的自然数, 它们的和为1995.试求a1 a2 + a2 a3 + ? ? ? + a9 a10 + a10 a1 的最小值. 解:将a1 , a2 , . . . , a10 按顺时针方向依次写在一个圆周上,于是所求表达式即为每相邻两数乘积的总和A. 并且将a1 , a2 , . . . , a10 的和记为N ,N 为S (N ). 先考虑这10个数为1, 2, . . . , 10的情况,即N = 55时. 不妨设a1 = 10,我们通过调整证明a1 , a2 , . . . , a10 依次为10, 1, 9, 3, 7, 5, 6, 4, 8, 2时取到最大值. 若aj = 1, j = 2,将(a2 , . . . , aj ?1 , aj )这一段整个的按逆过来的顺序排列,即变为(aj , aj ?1 , . . . , a2 ), 设操 作前所求表达式为A,操作后为A , 则A ? A = (10 + a2 aj +1 ) ? 10a2 ? aj +1 = (a2 ? 1)(aj +1 ? 10) (10, 1, 9, 3, 7, 5, 6, 4, 8, 2),且每次操作A的值都下降. ∴ S (55) = 10 + 9 + 27 + 21 + 35 + 30 + 24 + 32 + 16 + 20 = 224. 对于N & 55,显然最大的数大于10,第二大的数不小于9,. . . ,最小的数不小于1. 不妨设a1 最大,经类似的讨论可知道若将a1 , a2 , . . . , a10 从大到小依次以 10, 9, . . . , 1代替,必有按照上面 方式排列时取最小值. 设(b1 , b2 , . . . , b10 ) = (10, 1, 9, 3, 7, 5, 6, 4, 8, 2),ci = ai ? bi 0, c11 = c1 , b11 = b1 , b0 = b10 . 0,A的值下降了. 55.将和为N 的任意10个不同自然数所对应的表达式的最小值记同理若a10 = 2,通过类似的调整使a10 = 2,且A的值减少; 类似的经过有限次操作即得到(a1 , a2 , . . . , a10 ) =A = a1 a2 + a2 a3 + ? ? ? + a9 a10 + a10 a110 10 10=i=1bi bi+1 +i=1ci ci+1 +i=1ci (bi?1 + bi+1 )224 + (b2 + b10 )(N ? 55) = 3N + 59 而当(a1 , a2 , . . . , a10 ) = (N ? 45, 1, 9, 3, 7, 5, 6, 4, 8, 2)时,A = 3N + 59.∴ S (N ) = 3N + 59. 所以N = 1995时,a1 a2 + a2 a3 + ? ? ? + a9 a10 + a10 a1 的最小值为3 × 1995 + 59 = 6044.(0) 6.设n是大于1的奇数,已给 y0 = (x1 , x2 , . . . , xn?1 , xn ) = (1, 0, . . . , 0, 1). (0) (0) (0) (k?1) (k?1)设xi(k?1) (k?1)(k )=0 xi 1 xi= xi+1 时, i = 1, 2, . . . , n = xi+1 时, i = 1, 2, . . . , n(k?1)(k?1)其中xn+1 = x1 记yk =. = 1 , 2, . . .(k ) (k )(k ) (k ) (k ) (k ) (x1 , x2 , . . . , xn?1 , xn ), k若正整数m满足y0 = ym .求证:m是n的倍数. 证明:将一个圆周n等分,将x1 , x2 , . . . , xn?1 , xn 依次按顺时针方向写在这些分点上表示yk . 31(k ) (k ) 对于y0 ,显然它有唯一的对称轴,不妨设为竖直线. 因为xi(k )≡ xi(k?1)+ xi+1(k?1)(mod 2).所以将yk 中每两个相邻点上的数的和除以2的余数放在这段弧的中点上,再将原先的数撤去,显然对称轴π π 是不变的. 而再将它逆时针旋转 n 时,即得到yk+1 .所以yk+1 的对称轴是yk 的对称轴逆时针旋转 n .若y0 = ym ,它们的对称轴也相同,而中间变换了m次,对称轴旋转了 mπ n ,而它重合于竖直线, 所以它旋转 了π 的整数倍,记为kπ .∴mπ n= kπ, m = kn,即m是n的倍数.32 第十一届中国数学奥林匹克(1996年)天津 南开大学1. 设H 是锐角 ABC 的垂心, 由A向以BC 为直径的圆作切线AP ,AQ,切点分别为P ,Q. 求证:P, H, Q三点共线. 证明:设三条高的垂足分别为D, E, F ,BC 中点为O,P Q与AO交于R,则AO ⊥ P R. 由∠AP O = ∠ARP = 90? , AP O 由切割线定理AE ? AC = AP 2 . ∴ AO ? AR = AH ? AD. 若D与O重合,则H 与R重合,P, H, Q显然共线. 否则,O, D, H, R四点共圆,∠ORH = ∠ODH = 90? . ∴ AO ⊥ RH ,P, H, Q共线,证毕. 2.设S = {1, 2, . . . , 50},求最小自然数k ,使S 的任一k 元子集中, 都存在两个不同的数a和b,满足a + b整 除ab. 解:设a, b ∈ S ,满足a + b整除ab.设c = gcd(a, b),于是a = ca1 , b = cb1 ,其中a1 , b1 ∈ N,且gcd(a1 , b1 ) = 1.因 此 c(a1 + b1 ) = a + b|ab = c2 a1 b1 , a1 + b1 |ca1 b1 ∵ gcd(a1 + b1 , a1 ) = gcd(a1 + b1 , b1 ) = 1,所以a1 + b1 |c. 因为a, b ∈ S ,a + b 99,c(a1 + b1 ) 99.所以3 a1 + b1 9. 易知S 中所有满足a + b整除ab的不同数对共有23对如下: a1 + b1 = 3 : (6, 3)(12, 6)(18, 9)(24, 12)(30, 15)(36, 18)(42, 21)(48, 24) a1 + b1 = 4 : (12, 4)(24, 8)(36, 12)(48, 16) a1 + b1 = 5 : (20, 5)(40, 10)(15, 10)(30, 20)(45, 30) a1 + b1 = 6 : (30, 6) a1 + b1 = 7 : (42, 7)(35, 14)(28, 21) a1 + b1 = 8 : (40, 24) a1 + b1 = 9 : (45, 36) 令M = {6, 12, 15, 18, 20, 21, 24, 35, 40, 42, 45, 48}, |M | = 12.并 且 上 述23个 数 对 中 每 一 对 都 至 少 包 含M 中1个元素. 因此,若令T = S \ M ,则|T | = 38,且T 中任何两数都不满足题中条件,所以k 而下列12个满足题中条件的数对互不相交: (6,3)(12,4)(20,5)(42,7)(24,8)(18,9)(40,10)(35,14)(30,15)(48,16)(28,21)(45,36) 对于S 的任意一个39元子集R,只比S 少11个元素,而这11个元素至多属于上述12个数对中的11对,从而必 有一对属于R. 39. ∠ARP, AO ? AR = AP 2 . 又∠ADC = ∠BEC = 90? ,∴ H, D, C, E 四点共圆.∴ AE ? AC = AH ? AD.33 综上可知,所求的最小自然数k = 39. 3.设R为实数集合, 函数f : R → R适合条件 f (x3 + y 3 ) = (x + y )((f (x))2 ? f (x)f (y ) + (f (y ))2 ), x, y 为实数. 试证:对一切实数x,都有f (1996x) = 1996f (x). 证明:令x = y = 0,有f (0) = 0.令y = 0, f (x3 ) = x(f (x))2 . 所以f (x) = x 3 (f (x 3 ))2 , x ∈ R. 由此可知x 0时,f (x) 0;x 0时,f (x)1 1 1 10.设S = {a & 0|对于?x ∈ R, f (ax) = af (x)}. 显然1 ∈ S ,若a ∈ S ,由f (x) = x 3 (f (x 3 ))2 , ax(f (x))2 = af (x3 ) = f (ax3 ) = f (a 3 x)3 ) = a 3 x(f (a 3 x))2 . 所以(f (a 3 x))2 = (a 3 f (x))2 ,f (a 3 x) = a 3 f (x). 即a 3 ∈ S . 若a, b ∈ S ,则a 3 , b 3 ∈ S f ((a + b)x) = f ((a 3 x 3 )3 + (b 3 x 3 )3 ) = = = ∴ a + b ∈ S. 由1 ∈ S, 1 + 1 = 2 ∈ S ,由归纳法易知所有自然数n ∈ S . ∴ 1996 ∈ S ,即f (1996x) = 1996f (x). 4.8位歌手参加艺术会,准备为他们安排m次演出, 每次由其中4位登台表演.要求8位歌手中任意两位同 时演出的次数都一样多, 请设计一种方案,使得演出的次数m最少. 解:设任两位同时演出r次,则r ∴ 3|r, r 3, m 14.8 2 4 21 1 1 1 1 1 1 1 1 1 1 1 1 1(a 3 + b 3 )x 3 [(f (a 3 x 3 ))2 ? f (a 3 x 3 )f (b 3 x 3 ) + (f (b 3 x 3 ))2 ] (a 3 + b 3 )x 3 (a 3 ? a 3 b 3 + b 3 )(f (x 3 ))2 (a + b)x 3 (f (x 3 )2 ) = (a + b)f (x)1 1 1 1 1 2 1 1 2 111111111111=m,即14r = 3m.用1,2,. . . ,8代表8位歌手,如下14次演出满足要求: (1,2,3,4);(1,2,5,6);(1,2,7,8);(1,3,5,7);(1,3,6,8);(1,4,5,8);(1,4,6,7); (2,3,5,8);(2,3,6,7);(2,4,5,7);(2,4,6,8);(3,4,5,6);(3,4,7,8);(5,6,7,8). ∴ m = 14. 5.设n为自然数,x0 = 0, xi & 0, i = 1, 2, . . . , n且n n i=1xi = 1.求证:1i=1√xi π √ & . 2 1 + x0 + x1 + ? ? ? + xi?1 xi + ? ? ? + xn证明:设x0 + x1 + ? ? ? + xi?1 = cos θi ,则xi + xi+1 + ? ? ? + xn = 1 ? cos θi (i = 1, 2, . . . , n + 1). ∴ xi = cos θi+1 ? cos θi , θi ∈ [0, π 2 ]. 34 θ1 = 往证:π 2& θ2 & ? ? ? & θn & θn+1 = 0.n1i=1√cos θi+1 ? cos θi π √ & 2 1 + cos θi 1 ? cos θi π cos θi+1 ? cos θi & sin θi 2即 1n i=1而由sin θi ∈ (0, 1](i = 1, 2, . . . , n)n∴i=1cos θi+1 ? cos θi sin θin(cos θi+1 ? cos θi ) = cos θn+1 ? cos θ1 = 1i=1 π 2 ). n+θi 又由于 θi+1 & θi ,sin x & x(0 & x & 2∴i=1 ncos θi+1 ? cos θi sin θi+θi θi+1 2 sin θi+1 sin θi ?2 2 sin θi θi+1 2 sin θi sin θi ?2 sin θi=i=1 n i=1 n= 2i=1 nsinθi ? θi+1 2θi ? θi+1 2 i=1 π = θ1 ? θn+1 = 2 & 2 所以原不等式成立. 6.在 ABC 中,∠C = 90? , ∠A = 30? , BC = 1,求 ABC 的内接三角形(三顶点分别在 ABC 三边上的 三角形)的最长边的最小值. 解:令 DEF 为内接三角形,D, E, F 分别在BC, CA, AB 上. 对 于BC 上 的 任 意 一 点D,令∠EDF = 60? 保 持 不 变,设G, H 分 别 在AC, AB 上,∠ADG = ∠ADH = 60? .显然当E 从G运动到A时,F 从A运动到H . 因为∠DGA & ∠C = 90? ,所以DG & DA; 又∠DHA = ∠B + ∠BDH = ∠ADB & ∠C = 90? ,所以DA & DH . 所以必然存在某个E ,使得DE = DF ,即 DEF 为等边三角形.并且E 从G运动到A时,DE 严格增,DF 严 格减,所以有唯一的E ,使得 DEF 为等边三角形. 设BD = x,令CE = DE 2 = (1 ? x)2 + (√ 3 2 x, BF=1? x 2 .由余弦定理x 7 2 2 ? DF 2 = x2 + (1 ? x 2 ) ? 2x(1 ? 2 ) cos 60 = 4 x ? 2x + 1 √ 3 2 2 x) 2 =7 4 x ? 2x + 135 √ EF 2 = ( 3 ?√3 2 2 x)√ x x 7 2 2 ? + (1 + x 2 ) ? 2 3(1 ? 2 )(1 + 2 ) cos 30 = 4 x ? 2x + 1即DF = DE = EF ,此时 DEF 为等边三角形,即D固定时唯一的等边三角形.1 设DEF 为等边三角形,记AB, AC 中点分别为M, N ,BM 中点为S ,T 在AB 上,BT = 3 AB .D从B 运动到C 时,E 从C 运动到N ,F 从M 运动到S .设 DEF 的边长为a.7 2 4 2 3 x ? 2x + 1 = 7 a2 = 4 4 (x ? 7 ) + 7 .当x = 4 7 时,有最小值 考虑Z 在AB 上的位置, (1)Z 在AM 上,则ZX (2)Z 在BT 上,则ZY√ 3 2 BF √ 3 2 (13 7 .设此时在P QR的位置.3 7.下面证明对任意内接三角形XY Z ,最大边长不小于1 2 AC 2 3 BC √ √ 3 2 3 7; 3 7;= =&2 3&(3)Z 在M T 上,按照前面的方法作出正三角形DEF ,(F = Z ),x & 1 3, = ?x 2) &3 2 x= CE .则DE = DF = EF3 7;3 7.(3.1)X 在CD上,XZDF(3.2)Y 在CE 上,∠F EC & 90? , Y Z (3.3)X 在BD上,Y 在AE 上,XY = √EF3 7;CX 2 + CY 2√CD2 + CE 2 = DE3 7.3 7.综上所述, ABC 的内接三角形的最长边的最小值为36 第十二届中国数学奥林匹克(1997年)杭州 浙江大学1.设实数x1 , x2 , . . . , x1997 满足如下两个条件: √ 1 (1)? √ xi 3(i = 1, 2, . . . , 1997) 3 √ (2)x1 + x2 + ? ? ? + x1997 = ?318 312 12 试求x12 1 + x2 + ? ? ? + x1997 的最大值,并说明理由.解:考虑函数f (x) = (t + x)12 + (t ? x)12 ,(t为常数). 显然f (x)为偶函数且展开式中所有偶次项系数均为正,所有奇次项均为0,所以f (x)在[0, +∞)上为增函 数.12 1997, xi + xj 为定值时, 当|xj ? xi |的值越大时,x12 i + xj 越大. √ 1 这样我们逐次进行调整,过程如下:每次选取 ? √ & xi xj & 3,保持它们的和不变. 若xi + xj & 3 √ √ √ 1 1 1 3? √ ,则将(xi , xj )调整为(xi + xj ? 3, 3); 否则将(xi , xj )调整为(xi + xj + √ , ?√ ). 这样调整 3 3 3 √ 1 12 12 √ 后F = x12 1 + x2 + ? ? ? + x1997 的值增大,经过有限次这样的调整后, 最多有一个xi 不等于? 3 或 3,此所以对于?i, j, 1i&j时达到最大值. √ √ 1 1 设此时有k 个 3,1996 ? k 个? √ ,另一个为m ∈ [? √ , 3]. 3 3 √ √ 1 1 则k 3 + (1996 ? k )(? √ ) + m = ?318 3,m = √ (1042 ? 4k ). 3 3 ∴ ?1 1042 ? 4k 3, 1039 4 又因为k ∈ Z, ∴ k = 260, m = 3 3. √ 12 √ 1 12 2 ∴ F 260( 3)12 + 1736(? √ ) + ( 3) =
3 √ √ 2 1 当有260个xi 为 3,1个xi 为 3 3,其他均为? √ 时取等号. 3 2.点P 是凸四边形A1 B1 C1 D1 内一点, 且P 到各顶点的连线与四边形过该点的两条边的夹角均为锐角. 递推定义Ak , Bk , Ck 和Dk 分别为P 关于直线 Ak?1 Bk?1 ,Bk?1 Ck?1 , Ck?1 Dk?1 和Dk?1 Ak?1 的对称点 (k = 2, 3, . . .).考察四边形序列Aj Bj Cj Dj (j = 1, 2, . . .). 试问:(1)前12个四边形中,哪些必定与第1997个相似,哪些未必; (2)假设第1997个是圆内接四边形,那么在前12个四边形中,哪些必定是圆内接四边形,哪些未必. 解:设∠Dj Aj P, ∠Aj Bj P, ∠Bj Cj P, ∠Cj Dj P 分别为αj , βj , γj , δ ∠P Aj Bj , ∠P Bj Cj , ∠P Cj Dj , ∠P Dj Aj 分别为αj , βj , γj , δj . 不难知道Aj 为 Dj +1 Aj +1 P 的外心,所以αj +1 = 1 2 ∠Dj +1 Aj P = αj . 类似地,我们可以知道 (αj +1 , βj +1 , γj +1 , δj +1 ) = (αj , βj , γj , δj ) (αj +1 , βj +1 , γj +1 , δj +1 ) = (βj , γj , δj , αj ) ∴ (αj +4 , βj +4 , γj +4 , δj +4 ) = (αj , βj , γj , δj ) (αj +4 , βj +4 , γj +4 , δj +4 ) = (αj , βj , γj , δj ) ∴ Aj +4 Bj +4 Cj +4 Dj +4 Aj Bj Cj Dj 37 k √ 21043 4 又因为(αj +2 + αj +2 ) + (γj +2 + γj +2 ) = αj + γj + γj + αj = (αj + αj ) + (γj + γj ) 所以Aj +2 Bj +2 Cj +2 Dj +2 与Aj Bj Cj Dj 相应的对角和相等. 于是有(1)前12个四边形中,第1,5,9个必定与第1997个相似; (2)假设第1997个是圆内接四边形,那么在前12个四边形中,第1,3,5,7,9,11个必定是圆内接四边形. 下面说明对于前12个四边形中其他的四边形未必成立. 考虑筝形A1 B1 C1 D1 ,P 为其对角线交点,A1 B1 = A1 D1 , C1 B1 = C1 D1 , ∠B1 = ∠D1 & 90? .它不是圆内 接四边形. 设(α1 , β1 , γ1 , δ1 ) = (α, β, γ, δ ), 则(α1 , β1 , γ1 , δ1 ) = (α, δ, γ, β ). 所以(α2 , β2 , γ2 , δ2 ) = (α, β, γ, δ ); (α2 , β2 , γ2 , δ2 ) = (δ, γ, β, α),(β + δ & 90? ) ∴ ∠A2 = ∠D2 = α + δ, ∠B2 + ∠C2 = β + γ ,A2 B2 C2 D2 为等腰梯形,是圆内接四边形. 类似的可知A3 B3 C3 D3 为筝形,不是圆内接四边形.且∠A3 = ∠C3 = α + γ & 90? . A4 B4 C4 D4 为等腰梯形,是圆内接四边形. 这四个四边形互不相似,而四边形的形状显然是以4为周期变化. 所以在这个四边形序列中,前12个四边 形中,只有第1,5,9个与第1997个相似. 在上述序列中,以A2 B2 C2 D2 为第一个四边形,不难知道所有的第2k + 1个四边形均为圆内接四边 形, 而 其 他 四 边 形 均 不 是 圆 内 接 四 边 形,所 以 第1997个 是 圆 内 接 四 边 形,在 前12个 四 边 形 中,只 有 第1,3,5,7,9,11个是圆内接四边形. 综上所述(1)前12个四边形中,第1,5,9个必定与第1997个相似,其他未必; (2)假设第1997个是圆内接四边形,在前12个四边形中,第1,3,5,7,9,11个必定是圆内接四边形,其他未必. 3.求证存在无穷多个正整数n,使得可将1, 2, . . . , 3n列成数表 a1 b1 c1 满足如下两个条件: (1)a1 + b1 + c1 = a2 + b2 + c2 = ? ? ? = an + bn + cn 且为6的倍数; (2)a1 + a2 + ? ? ? + an = b1 + b2 + ? ? ? + bn = c1 + c2 + ? ? ? + cn 且为6的倍数.1 证明:显然6n| 2 (3n)(3n + 1), 18| 1 2 (3n)(3n + 1).a2 b2 c2. . . an ... ... bn cn必有n ≡ 1 (mod 4), n ≡ 0 (mod 3),所以n ≡ 9 (mod 12). 下面Ai 中第1,2,3行分别记为α(i), β (i), γ (i),且α(i) + k 表示将α(i)中每个数都加上k ,其他类似. 先构造A9 满足条件:设 ? ? ? A3 = ? 5 ? 9 1 8 3 4 6 ? ? ? 7 ? ? 238 显然各行各列之和均为15,令 ? ? ? A9 = ? β (3) + 9 ? γ (3) + 18 α(3) β (3) + 18 γ (3) α(3) + 9 γ (3) + 9 ? ? ? α(3) + 18 ? ? β (3)易知A9 中的27个元素为1,2,. . . ,27,并且各列之和为15 + 9 + 18 = 42 ≡ 0 (mod 6); 各行之和为3(15 + 9 + 18) = 126 ≡ 0 (mod 6).所以9是满足条件的正整数. 设m满足条件,且形成的数表(矩阵)为Am ,各行之和为6u,各列之和为6v . 构造A3m 如下: ? ? ? A3m = ? β (m) + 3m γ (m) α(m) + 6m ? γ (m) + 6m α(m) + 3m β (m) α(m) β (m) + 6m γ (m) + 3m ? ? ? ? ?则A3m 中9m个元素为1, 2, . . . , 9m,并且各行之和为18u + 9m2 ,各列之和为6v + 9m. 构造A9m 如下: ? ? ? A9m = ? β (3m) + 9m ? γ (3m) + 18m α(3m) β (3m) + 18m γ (3m) α(3m) + 9m γ (3m) + 9m ? ? ? α(3m) + 18m ? ? β (3m)则A9m 中27m个元素为1, 2, . . . , 27m,并且各行之和为54u + 108m2 ≡ 0 (mod 6), 各列之和为6v + 36m ≡ 0 (mod 6). ∴ 9m也是满足条件的正整数,由归纳法不难证明对?k ∈ N? , 9k 是满足条件的正整数,显然有无穷多个. 4.四边形ABCD内接于 O,其边AB 与DC 的延长线交于点P , AD与BC 的延长线交于点Q,过Q作该圆 的两条切线QE 和QF , 切点分别为E, F .求证:P, E, F 三点共线. 证明:连接P Q,并且在P Q上取一点M ,使得B, C, M, P 四点共圆,则QE 2 = QM ? QP = QC ? QB , 并且∠P M C = ∠CBA = ∠P DQ.所以C, D, Q, M 四点共圆. 所以P M ? P Q = P C ? P D. P Q2 = P M ? P Q + QM ? P Q = QC ? QB + P C ? P D. 连接P F ,设P F 与圆的另一交点为E ,作QG ⊥ P F ,垂足为G. 则P D ? P C = P E ? P F, QF 2 = QC ? QB . 所以P E ? P F + QF 2 = P Q2 ,即P E ?}

我要回帖

更多关于 设1 a bi b ai 的文章

更多推荐

版权声明:文章内容来源于网络,版权归原作者所有,如有侵权请点击这里与我们联系,我们将及时删除。

点击添加站长微信